Главная Юзердоски Каталог Трекер NSFW Настройки

Наука

Ответить в тред Ответить в тред
<<
Назад | Вниз | Каталог | Обновить | Автообновление | 507 60 121
Тупых вопросов тред очередной Аноним 17/01/23 Втр 23:58:39 570861 1
image.png 11526Кб, 3000x1815
3000x1815
Аноним 18/01/23 Срд 04:05:45 570863 2
Люди в среднем сколькиюродные родственники друг другу? Генеалогические древа в целом изолированны друг другу или смешиваются? Ну т.е. вероятность случайно оказаться 10-15юродным братом прямых потомков цезаря и чингисхана минимальна, например?
Аноним 18/01/23 Срд 04:07:48 570864 3
> Генеалогические древа в целом изолированны друг другу или смешиваются? Ну т.е. вероятность случайно оказаться 10-15юродным братом прямых потомков цезаря и чингисхана минимальна, например?
Бля, хуйню сказал, лучше так: если взять 200 рандомов и установить родственные связи со мной, то степень -юродности будет сильно скакать (от 5 до 200, например) или бОльшая часть будет мне 20-30юродными?
>>570863
Аноним 18/01/23 Срд 09:44:05 570873 4
Что самое главное нужно, для качественного понимания термодинамики и статистической физики на 2 семестре? Конкретно интересует теория вероятности что там нужно познать?
Аноним 18/01/23 Срд 09:53:48 570874 5
c7526c366.png 155Кб, 1685x1289
1685x1289
Поясните за интерференцию одного фотона. Вот если их, фотонов, несколько, то картинка полосатая. А если он один, то откуда там полосы? Я уже видел такие мнения:
1. Фотон оставляет за собой фантомный след, с которым взаимодействуют другие фотоны.
2. Вместе с фотоном летят фотоны из параллельной вселенной.
3. Выпустить один фотон невозможно, их там как минимум несколько.
4. Интерференция одного фотона - миф и развод.
Аноним 18/01/23 Срд 12:03:21 570879 6
>>570874
>1. Фотон оставляет за собой фантомный след, с которым взаимодействуют другие фотоны.
Вот этот не плох, намного лучше моей предыдущей теории.
Аноним 18/01/23 Срд 13:31:43 570883 7
>>570874
Там пускают много фотонов, но по одному, пока суммарно они не отрисуют полосы.
Аноним 18/01/23 Срд 14:17:00 570885 8
>>570883
А почему ни один из фотонов не попадает на темные области? на самом деле попадает но гораздо реже
Аноним 18/01/23 Срд 14:17:24 570886 9
>>570873
Понимания общей физики на уровне школы, лол. Или хотя бы отличать макро от микро состояний и что вообще такое квазистатические процессы.
Теория вероятностей опять тоже почти на уровне школы. Единственное сложное это вывод распределения, ибо там предел особым образом надо брать.
Аноним 18/01/23 Срд 14:44:07 570887 10
image.png 178Кб, 2825x1537
2825x1537
Что не так с этой задачей? Есть коробка с слоем воды внутри, а сверху вакуум. Расписываем ситуацию равновесия, когда вода не выливается, тогда найдем высоту h, при которой атмосферное давление сможет держать слой воды. Ответ 100 метров. Что я не так сделал?
Аноним 18/01/23 Срд 15:59:07 570889 11
16719614053300.mp4 6810Кб, 1280x720, 00:02:03
1280x720
Что такое умиление у человека с эволюционной точки зрения? Что это за чувство/ощущение, почему оно возникает? Почему смотря на вебмрелетейд начинаешь умилиться и чувствовать мимими если ты не ебнутый психопат-живодер?
Аноним 18/01/23 Срд 16:55:28 570890 12
image.png 1840Кб, 910x1213
910x1213
image.png 1079Кб, 880x736
880x736
image.png 3916Кб, 1280x1920
1280x1920
image.png 649Кб, 800x600
800x600
у коптыных животных есть КОПЫТА
Неважно, свинья это, лошадь, козёл или лось - ни все копытные.
Но вот такой вопрос: под какой биом заточены копыта и на какой поверхности дают преимущество?
Я подумал так: если копыта дают преимущество в горах, где например живут горные козлы, которые ходить и бегать по самым отвесным скалам могут, то в пустыне у верблюдов копыта другого толка, нацеленные на то, чтобы не тонуть в песке, широкие ка у лося, но верблюда и скорость маленькая, а лось-то побыстрее будет, но у него и биом в целом с более твёрдой почвой. А где лучше?
Человеческая нога для чело лучше приспособлена? К земле и глине? А птичья лапа, куриная ли, гусиная ли, орлиная ли? А лапа какой-нибудь кошки, рыси или тигра? А у обезьян, будь то орангутан, горилла, игрунка или лемур?
Можно ли по ноге животного узнать, в каком биоме оно обитает? Почему нет зверей, у которых вместо лап какие-нибудь щупальца-слизняки, большие такие мясистые слизнеподобные похожие по форме на лист дерева отростки?
Аноним 18/01/23 Срд 17:18:38 570892 13
>>570885
>>570874
Тебе уравнения квантовой механики не подходят как объяснение?
>>570873
Матан должен быть на нужном уровне, ты должен частными производными пользоваться так же легко, как пользуешься сейчас умножением, чтобы замена переменной дифференцирования, для тебя было не сложнее раскрытия скобок в (a+b)2
Ну и как анон выше написал, интегралы многомерные и несобственные надо уметь и брать, и понимать, что там с предельными переходами. Поэтому Фихтенгольц + Задачник Демидовича (желательно прям порешать его много, т.е. больше 70% заданий) + м.б. Зорич после
Аноним 18/01/23 Срд 18:30:23 570896 14
>>570892
>Задачник Демидовича
Что нужно знать, чтобы решать оттуда задачи? Я остановился в развитии примерно на квадратных уравнениях с параметрами, но все еще могу вспомнить, как решать степенные уравнения, показательные. + еще с первого курса помню правило Лопиталя для нахождения предела, но матан как таковой уже вообще весь из головы выветрился.
Аноним 18/01/23 Срд 19:26:13 570901 15
Пиздец, что за пидарас удалил тред?

В архиве тоже ничего нет
Аноним 18/01/23 Срд 20:27:26 570903 16
>>570889
Это родительский инстинкт.
Аноним 18/01/23 Срд 20:55:54 570904 17
>>570903
А почему я не ощущаю такие же эмоции, если вижу детенышей человека?
Аноним 18/01/23 Срд 21:39:45 570906 18
Есть вопрос касательно кандидатской степени, дваченые. Вот учился я на, скажем, математика, оттуда плавно перетекает в айти и в этой сфере готовлю кандидатка, но к чистой матеше не охладел и хочу себе, значит, степень кандидата ф. -м. наук. Что нужно, чтобы стать соискателем и насколько это вообще реалистичная задача? Закрыл ли я себе пути в физмат науку?
Аноним 18/01/23 Срд 21:47:14 570907 19
>>570896
>Фихтенгольц
Читай его, темы в задачнике есть прям в названии глав
Аноним 18/01/23 Срд 23:17:40 570908 20
Аноним 19/01/23 Чтв 11:46:04 570919 21
>>570861 (OP)
Ребят как теоретически рассчитать какими свойствами будет обладать сплав иридия и вольфрама?
Аноним 19/01/23 Чтв 13:40:44 570924 22
>>570896
Изучай матан, забей на школьные темы. Все что надо вспомнишь. Теорию изучай, но сильно не зацикливайся, больше старайся решать. Введи в поиск мфти программа там найдешь листочек по каждому предмету, где будут книжечки, темы и задачи. Основывайся на этом.
Аноним 19/01/23 Чтв 14:19:31 570925 23
Аноним 19/01/23 Чтв 14:40:37 570927 24
>>570892
Спасибо. А что на счет теории вероятности? Я все время читал статическая физика и гадал, что там статичного?? Но недавно понял, что читается статистическая логично там нужна теория вероятности и статистика. А что конкретно?
Аноним 19/01/23 Чтв 16:44:22 570929 25
Аноним 19/01/23 Чтв 17:40:45 570930 26
>>570927
Ну сначала матан прокачиваешь, а потом можешь просто лекции посмотреть на ютубе. Посмотри один семестр лекций 20, по 1.5 часа с конспектированием и всё будет норм.
Аноним 20/01/23 Птн 11:02:40 570963 27
Почему чтобы изучать науку нужно быть поехавшим?
Аноним 20/01/23 Птн 12:38:20 570968 28
>>570963
Это заблуждение, придуманное быдлом, чтобы успокаивать себя, и чтобы мешать детям идти в науку.
Прецессия северного полюса Земли fat_alien 20/01/23 Птн 20:34:57 570983 29
Прецессиясеверн[...].png 521Кб, 1136x1136
1136x1136
Npolesqare30deg[...].png 595Кб, 1136x1136
1136x1136
Уникальное изображение для 2ч. такого нет во всём интернете! Доказательство — https://en.wikipedia.org/wiki/Axial_precession

Проекция пути северного полюса Земли на неподвижное небо эпохи J2000.0 при лунно-солнечной и планетной[1] прецессии для интервала времени от 48000 г. до н. э. до 52000 г. н. э. В центре показан путь прецессии северного полюса орбиты Земли для того же времени и белым крестом обозначено направление северного полюса инвариантной плоскости[2]. Голубой сеткой обозначена эклиптическая система координат и жёлтой — экваториальная система координат для эпохи J2000.0. Для собственного вращения Земли зависимости ψA и ωA, для эклиптики — PA и QA из статьи [3]

1. Бакулин П.И. Курс общей астрономии. — 4-е изд.. — М.: «Наука», 1977. — 544 с.
2. D. Souami, J. Souchay. The solar system’s invariable plane (англ.) // Astronomy & Astrophysics. — 2012-07-01. — Vol. 543. — P. A133. — ISSN 1432-0746 0004-6361, 1432-0746. — doi:10.1051/0004-6361/201219011.
3. J. Vondrák, N. Capitaine, P. Wallace. New precession expressions, valid for long time intervals (англ.) // Astronomy & Astrophysics. — 2011-10-01. — Vol. 534. — P. A22. — ISSN 1432-0746 0004-6361, 1432-0746. — doi:10.1051/0004-6361/201117274.
Аноним 21/01/23 Суб 09:51:21 571015 30
>>570861 (OP)
Удалённый тред не сохранился?
Аноним 21/01/23 Суб 10:45:08 571017 31
>>570874
Очевидно, если щели близко расположены и через одну цель пролетает фотон, то через две щели пролетает окружающее фотон его поле. Поле интерферирует, а фотон отклоняется полем в соответствии с картиной интерференции.
Аноним 21/01/23 Суб 14:06:54 571027 32
>>571017
Что ещё за поле фотона? Фотон сам и есть кусок поля.
Аноним 21/01/23 Суб 15:31:31 571032 33
>>570890
>Но вот такой вопрос: под какой биом заточены копыта и на какой поверхности дают преимущество?
Твердая и сухая почва саванн где собственно копытные и появились.
>Человеческая нога для чело лучше приспособлена?
Человеческая нога это переход от лазанья по деревьям всеми четырмя конечностями к хождению по земле исключительно на двух ногах.
>куриная
К рытью земли
>гусиная
К плаванью
>орлиная
К сидению на ветке и хватанию отбивающегося пропитания.
>Конечности ... похожие по форме на лист дерева отростки
Такие звери есть - китообразные и ластоногие.
Аноним 21/01/23 Суб 15:58:43 571034 34
>>570874
> А если он один, то откуда там полосы? Я
Там и нет полос. Один фотон даст точку
Аноним 21/01/23 Суб 16:27:55 571035 35
>>571027
Я про что и говорю. Фотон сделан из полей. У него прост структура есть. Есть уплотненные маленькие части, а есть внешние пространные оболочки. Через одну щель пролетают мелкие плотные частички фотона, а через обе пролетают протяженные внешние оболочки. Оболочки распидорашиваются об щели и за щелями получается интерференционное распидорашенное поле из внешних оболочек фотона. А сами частички попав в это поле испытывают отклонение и летят уже в экран. Почитай что нибудь про волну-пелота.
Аноним 21/01/23 Суб 16:56:49 571036 36
>>571035
Фотон не сделан из полей. Это поле сделано из фотонов.
Аноним 21/01/23 Суб 18:22:56 571039 37
Два вопроса. Если в литий ионных аккумуляторах, литий заменить на цезий какие его характеристики изменяться? Если залить в кислотный аккумулятор хлорной кислоты вместо серной, этот электролит будет сильнее?
Аноним 22/01/23 Вск 00:19:59 571043 38
>>571039
Не трогай литий в ионных аккумуляторах. Литий при контакте с воздухом горит. По миру пойдёшь, если жив останешься.
>>571035
>Фотон сделан из полей
Фатон не сделан из полей, не более чем фонон сделан из звуков. Фотон это состояние электромагнитного поля.
>У него прост структура есть.
У него нет структуры.
>Через одну щель пролетают мелкие плотные частички фотона,
Фотон это волна, но по причине того, что на квантовом уровне оперирует математика его представляют как частицу. Двойственность фотона это математическая приблуда которая упрощает жизнь физикам, как и допущение, что у него нету массы. Все это утверждения делаются в рамках теории. А задача теории далать успешные предсказания.И если факты не опровергают утверждение теории и она способна делать предсказания то теория принимается и используется. Это причиной того почему фактически зная что электрон движется от минуса к плюсы мы все ещё полагаем что он движется от плюса к минусы. Наука немного сумашедшая, но так надо.
Аноним 22/01/23 Вск 07:07:26 571048 39
>>571043
>зная что электрон движется от минуса к плюсы мы все ещё полагаем что он движется от плюса к минусы
Почему знаки просто не поменяли волевым решением, как в некоторых странах меняли стороны дорожного движения?
Аноним 22/01/23 Вск 12:07:18 571049 40
>>571043
>Не трогай литий в ионных аккумуляторах. Литий при контакте с воздухом горит. По миру пойдёшь, если жив останешься.

И это твой ответ, я тебя про цезий спрашивал, лучше бы просто промолчал тогда. Металлический литий тока в литиевых батарейках, в аккумуляторах он в виде оксида а не в виде металла.
Аноним 22/01/23 Вск 14:06:14 571050 41
Проект ИТЕР - реальность или развод?
И если это не взлетит - какие проблемы возникнут на этом фоне?
Аноним 22/01/23 Вск 14:07:59 571051 42
>>571048
Зачем? + и - это не белее чем обозначения. Например в электростатических генераторах лейденовские банки используются потому как сторона с + и с - определяется абсолютно рендомно. Если ты подключешь к подобной машине обычный кандер он рванет нафиг из за неправильной полюсовки.
Аноним 22/01/23 Вск 19:28:31 571082 43
>>570874
Мне больше интересно, как в 1803 году Юнгу удалось прорезать щели, по масштабу сопоставимые с длиной волны (700нм)?
Аноним 23/01/23 Пнд 00:57:33 571093 44
>>571082
нахуя прорезать? два лезвия возьми, одно зафиусируй, второе коси вращения прикрепи, вращая ось можно сколь угодно близко подойти к первому лезвию
Аноним 23/01/23 Пнд 01:38:55 571094 45
>>571050
Проект то вполне реальный, только вот ученые изнасиловали журналистов, что породило много всяких мифов.
Главная цель ИТЕР это изучения физики плазмы и проверки туевой хучи моделей. Тут целое не паханное поле, ибо на компьютерах это говно считается еще хуже чем классические ньютоновские жидкости, поэтому ждут экспериментальных данных. Когда ИТЕР введут в строй, то первые время он вообще будет в режимах очень далеких от заветное термояда.
Аноним 23/01/23 Пнд 02:34:32 571095 46
>>571093
Нихуя не понял. Можешь нарисовать?
Аноним 23/01/23 Пнд 10:48:16 571102 47
>>571095
Возьми две идеально ровные плоскости, положи между ними пылинку размером 1 микрон, и получишь щель 1 микрон.
Маленькие плоскости очень легко и быстро получаются с неровностью 0.1 микрон по методу трёх поверхностей полированием взаимным.

Но другое дело что у всех получается этот опыт если даже тупо в картонке прорезать лезвием щели.
Аноним 23/01/23 Пнд 18:00:28 571118 48
14516352508660.gif 374Кб, 220x219
220x219
>>571043
>Фотон это состояние электромагнитного поля.
Он состоит из этого поля.
>У него нет структуры.
Есть. Взгляни хотя бы на этот график. Из него видно что фотон это сцепленная система частиц. Частица-античастица-частица-античастица и т.д. Поля частиц складываются, поэтому кажется что фотон не обладает зарядом.
>Двойственность фотона это математическая приблуда
Это система из мелких частиц. Значит в целом это частица. Но частиц много и сцеплены они частица-античастица-частица-античастица, поэтому при пролете мимо детектора поля от этих частиц будут измеряться как электромагнитная волна.
Аноним 23/01/23 Пнд 18:37:11 571120 49
16092541035150.gif 2006Кб, 300x225
300x225
Аноним 23/01/23 Пнд 19:51:44 571124 50
Какие из доступных прозрачных материалов пропускают больше всего по энергии солнечного спектра какой спектр у поверхности земли?
Обычное стекло хорошо пропускает видимый и полностью отрезает инфракрасный
Чисто кварцевое стекло дорого слишком

Какие-то материалы может равномерно и видимый и инфракрасный пропускают, но например хоть и равномерно но настолько хуже, что по знергии пропустится меньше чем стекло обычное, которое только видимый пропускает и почти полностью отрезает инфракрасный.
Аноним 23/01/23 Пнд 19:54:00 571125 51
Как бы найти нормально хорошие графики пропускания спектра для полиэтиленовой плёнки, поликарбоната,
Ещё интересно наверное рукава для запекания делают из какого-то особенного материала, раз такой термостойкий

Других доступных в свободной продаже, а значит доступных прозрачных материалов я не знаю.
Аноним 24/01/23 Втр 19:07:57 571175 52
>>571094
>ибо на компьютерах это говно считается еще хуже
Подробнее
Аноним 24/01/23 Втр 19:15:40 571178 53
>>571124
>какой спектр у поверхности земли?
AM1 AM1.5 AM3
>Какие из доступных прозрачных материалов пропускают больше всего по энергии солнечного спектра
воздух
>полностью отрезает инфракрасный
полностью со скольки микрон начинается?
>поликарбоната
https://refractiveindex.info/?shelf=organic&book=polycarbonate&page=Zhang

только учти, что видов полимером как ёбырей у твоей еот
Аноним 24/01/23 Втр 22:14:37 571184 54
>>571178
>полностью со скольки микрон начинается?
С 1 где-то.
>воздух
Воздух это не материал как стекло или плёнка, это среда

>https://refractiveindex.info/?shelf=organic&book=polycarbonate&page=Zhang
Чёт не очень график какой-то, в видимом спектре и ближе к нему не видно что там, и что значит он вообще, типа самый пик прозрачности на длине волны этот конкретный материал пропускает 62% излучения с длиной волны 8.2 микрометра?
Аноним 24/01/23 Втр 22:32:16 571185 55
>>571184
Определение среды и материала какое?
>Чёт не очень график какой-то, в видимом спектре и ближе к нему не видно что там,
Увеличь
>типа самый пик прозрачности на длине волны этот конкретный материал пропускает 62% излучения с длиной волны 8.2 микрометра?
нихуя не понял
Аноним 24/01/23 Втр 22:47:08 571186 56
>>571185
>нихуя не понял
Ну я типа не понял что там за хуйня, одна линия где-то в районе полтора, другая от 0 до 0.6 где-то
Я знаю 0 это ноль пропускание, 1 это полное пропускание, или от 0 до 100% тоже знаю.
А что там за хуйня в графиках которые ты скинул, в районе полтора, и от 0 до 0.6, но почти всегда возле нуля, я не понял.
Типа он ничего в инфракрасном почти не пропускает кроме небольшого пика с максимальным пропусканием 60% на длине волны примерно 8.2 микрометра?
Ты сам график хоть смотрел

>Определение среды и материала какое?
Держит форму под собственной массой хотя бы, газ и жидкость не материал. И примеры приведены что примерно нужно, в виде чего-то похожего на листы стекла или плёнку.

>Увеличь
Там максимум до 2.5 мкм
Аноним 24/01/23 Втр 22:55:06 571187 57
Screenshot 2023[...].png 16Кб, 435x411
435x411
Screenshot 2023[...].png 28Кб, 747x394
747x394
>>571186
>Там максимум до 2.5 мкм
нет. до 0.4
>Держит форму под собственной массой хотя бы,
Жидкий металл не матераил?
>Ну я типа не понял что там за хуйня, одна линия где-то в районе полтора, другая от 0 до 0.6 где-то
Ты куда блядь смотришь? Там снизу специально есть окно с пропусканием. Или ты на графики n, k только смотришь?
Аноним 25/01/23 Срд 11:16:35 571211 58
>>571187
>Жидкий металл не матераил?
Нет, ведь из него нельзя ничего сделать, а только залить в форму из материала.
При этом что-то держащее нагрузку типа грунта или бетона может быть и средой и материалом, но то что нагрузку не держит хоть и является средой а материалом быть не может.
>Ты куда блядь смотришь?
Охуеиь, там и снизу что-то есть оказывается, а то я думаю что это за хуйня
Ну супер, если энергетический максимум на 550нм где-то, то там на тех длинах где у него провплы в прозрачности уже никакой энергии не будет почти.

А можешь объяснить что тут вообще ещё кроме этого на этой странице за данные такие есть кроме прозрачности, вижу отражение нескольких видов, а что за n k
Аноним 25/01/23 Срд 11:44:36 571213 59
>>571187
По отражению ещё три таких вопроса
Первое, отражение плюс пропускание больше единицы, это значит прозрачность смотрят по тому что уже гарантированно не отразилось а именно вошло в материал, и смотрят поглотилось ли оно внутри или нет?

Второе, вот смотрю я медь например. Как они получили не строго ноль прозрачности при видимом спектре? Да, там минус 27 степень например... Но не строго ноль. Ладно бы это была какая-то апроксимация, но там довольно сложной формы график именно на этом интервале длин волн, а не просто линия соединяющая две точки вне рассматриваемого диапазона.

И третье, там можно как-то получить график отражения с длинами волн по оси икс а не по углу? Мне разный угл на графике не нужен, всегда 0 надо.
Аноним 25/01/23 Срд 11:45:23 571214 60
Аноним 25/01/23 Срд 12:07:44 571216 61
Аноним 25/01/23 Срд 12:33:21 571219 62
>>571216
И чё? Как была просто среда так и есть и на этих видео.
Суть-то среды и есть что её можно крутить как угодно вот такими штуками.
Аноним 25/01/23 Срд 13:22:35 571222 63
>>571175
Жидкости и газы описываются схожими дифференциальными уравнениями в частных производных. А они решаются довольно хреново.
Для плазмы еще добавляются уравнения максвелла. Добавление к системе уравнение еще одной увеличивает сложность экспоненциально.
Аноним 25/01/23 Срд 16:27:11 571228 64
Есть двигатель от мотоблока на 7 л.с. и максимальными оборотами 3500. Как посчитать самый эффективный генератор переменного тока для такого движка?
Аноним 25/01/23 Срд 17:59:57 571249 65
>>571219
гондон тоже можно крутить как угодно. гандон - среда?
>>571213
>Первое
там расчетное значение для пропускания а не измеренное, и три варианта расчета.
>Как они получили не строго ноль прозрачности при видимом спектре?
из-за представлений чисел в компуктере скорее всего
+там где-то плазменная частота болтаться будет
> там можно
нет, вроде
>>571211
> а что за n k
показатель преломления и поглащения
Аноним 25/01/23 Срд 18:26:31 571253 66
image.png 13Кб, 1028x741
1028x741
Этот график в P V координатах показывает наборы значений состояний идеального газа (кроме температуры, она может быть любая в любой точке). И как я понимаю, он показывает множество наборов состояний равновесия в виде линий? Все состояния равновесия идеального газа с некоторым объемом показывают красные вертикальные линии, все состояния равновесия с некоторым давлением все синии горизонтальные, а все состояния равновесия с некоторой температурой ветри гиперболы (чем выше гипербола, тем выше температура)?
Аноним 25/01/23 Срд 18:45:45 571255 67
>>571228
Нужно для расчёта знать собираешься ли ты ставить диодный мостик.
Аноним 25/01/23 Срд 18:47:58 571256 68
>>571249
>гондон тоже можно крутить как угодно. гандон - среда?
Нет, ведь он нагрузку держит, а газ и жидкость нет.
Аноним 25/01/23 Срд 19:12:22 571258 69
Опыт с бочкой паскаля - если столб вверх толшиной 0.1мм всего, то работа по подъёму и закачке воды в него нужна мизерная, несопоставимая с энергией разрыва/растяжения бочки же? А если в бочке дырку сделать, из неё хлынет вода с такой силой, что хоть сталь разрежет? Или как только вода польётся, то связь с верхним стобом воды потеряется и давление уйдёт?
Аноним 25/01/23 Срд 19:17:43 571259 70
>>571253
>Все состояния равновесия идеального газа с некоторым объемом показывают красные вертикальные линии, все состояния равновесия с некоторым давлением все синии горизонтальные
Да, только это тривиально и бессмысленно. Потому что через каждую из этих прямых проходит бесконечное число гипербол, соответствующих разной температуре. Что из этого можно получить? Все равно что сказать каждой точке в плоскости графика соответствует какое-то состояние. Ну да, соответствует, можно сделать так, чтобы в баллоне емкостью 1 литр было давление 10 атм. Можно сделать так, чтобы в баллоне емкостью 1 литр было давление 15 атм. Можно сделать так, чтобы в баллоне емкостью 20 литров было 15 атм. Дальше что?
Единственный смысл там в гиперболах, которые показывают как меняется 1 конкретное давление при изменении 1 конкретного объема.

А вот тебе вопрос на понимание. Сколько гипербол может проходить через точку, которую ты нарисовал на своем графике. И почему именно столько.
Аноним 25/01/23 Срд 19:22:34 571260 71
>>571258
>как только вода польётся, то связь с верхним стобом воды потеряется и давление уйдёт?
Да
Аноним 25/01/23 Срд 19:31:47 571261 72
>>571258
>Или как только вода польётся, то связь с верхним стобом воды потеряется и давление уйдёт?
Связь не потеряется, но как только вода польется, уровень столба начнет уменьшаться. Давление хоть сталь режь будет только в первую миллимикросекунду. Сколько там воды в твоем 0,1 мм столбе? Первая вытекающая снизу капля будет иметь туже энергию, как будто она упала с высоты столба, дальше - меньше. И вся общая энергия в точности равна энергии, потраченной на закачку воды в столб. Соответственно расиянуть бочку этот столб тоже сильно не сможет, как только бочка хоть чуть чуть растянется вода из столба уйдет вниз, потому что бочка растянулась и ее объем увеличился. Можно долевать сверху воды, но опять в сумме потраченная на это эгергия будет равна энкргии разрыва бочки.
Короче, это такой гидравлический аналог рычага. Можно маленькой силой двигать длинный конец на большое расстояние, а можно напрячься и передаинуть короткий конец на миллиметр, но работа и так и так будет одна и таже.
Аноним 25/01/23 Срд 20:12:15 571263 73
>>571256
>Нет, ведь он нагрузку держит, а газ и жидкость нет.
схуев? шар из воды в космосе будет держать нагрузку
Аноним 25/01/23 Срд 20:39:50 571267 74
>>571256
В видосах феррожидкость(вода с наночастицами железа). Частицы железа реагируют с электро-магнитным полем и в результате вода начинает "втекать в очертания полей". То есть здесь работает два фактора поверхностное напряжение жидкости и мощность электромагнитных полей. Это по сути как контейнер где стенки силовое поле.
Аноним 25/01/23 Срд 21:46:06 571274 75
>>571263
Ну и что ты из него сделать сможешь
Аноним 26/01/23 Чтв 17:06:50 571334 76
>>571259
Если не менять количества вещества то через 1 точку проходит 1 гипербола, соответствующая некоторой температуре, а значит в этой точке изображено некоторое одно из возможный состояний равновесия. Если менять количество вещества, то да, там может оказаться любая гипербола. То есть в принципе существуют состояния равновесия ну совершенно произвольные (для идеального газа). Хоть 110000 кельвинов, 1м^3 и давлением чуть ниже земного. Только вот количество вещества, там, конечно, будет мизирное.
Если зафиксировать кол-во вещества, то для 110000 кельвинов будет какая-то гипербола очень-очень высоко, на которой будут все состояния равновесия.
Особенно полезно пространство состояний в том, что равновесные процессы в них будут в виде линий, а изо процессы и адиабатические еще и в виде красивых. Легко можно понять, к примеру, что охладившись, нельзя прийти на ту же температуру адиабатическим расширением, а только сжатием, тупо потому что адиабата быстрее прижимается к 0 при расширении ( в PV )
Аноним 26/01/23 Чтв 21:26:21 571339 77
>>571334
Именно. Так становится ещё нагляднее почему
>Все состояния равновесия идеального газа с некоторым объемом показывают красные вертикальные линии, все состояния равновесия с некоторым давлением все синии горизонтальные
не имеет смысла. Потому что даже одна точка может соответствовать бесконечному количеству состояний различных систем. А смысл графика - поеазать изменения в 1 конкретной системе.
Аноним 26/01/23 Чтв 22:47:49 571342 78
А в отличии от горячей относительно разреженной(атмосферное давление) плазмы, если попытаться разрушить вещество оставляя его в холодном состоянии, чисто сжимая(компенсируя нагревание при сжатии, давая остыть) то хуй что получится буквально до каких-то ебанутых значений?

Т.е. походу получается что буквально до нейтронной звезды будет вполне себе обычное вещество?
Аноним 27/01/23 Птн 10:17:46 571377 79
Существуют ли на планете организмы, которые выделяют не СО2, а СО?
Аноним 28/01/23 Суб 00:42:05 571413 80
Какие теории/догадки ОФИЦИАЛЬНЫЕ более-менее общепринятые есть о том что стабилизирует от распада нейтрон в ядре?

По-идее же нейтрон к нейтрону притягивается точно так же как протон к нейтрону.
Значит стабилизирует от распада или гравитация или сильные силы или они вместе.
Но в любом случае и то и то должно при связке нейтрон+нейтрон быть таким же как при нейтрон+протон, даже возможно получше.

Но насколько знаю в природе не встречается свободных дейтронных ядер(электрон-то им не нужен, и атома не прлучится, и плазмой это не будет даже с учётом что без электрона)

А что если попытаться на скорость слепливать как можно больше нейтронов до того как они через 15 минут полураспадутся?
Аноним 28/01/23 Суб 01:07:41 571415 81
>>571342
Белые карлики состоят из электронно-ядерной плазмы. Ничего обычного в ней нету.
Аноним 28/01/23 Суб 01:16:22 571416 82
>>571413

>По-идее же нейтрон к нейтрону притягивается точно так же как протон к нейтрону.
Физики полагают стабильной конфигурацию из трёх нейтронов. Даже есть попытки назвать данное образование химическим элементом. Разумеется всем похуй что до сих пор это чистая теории и шансы на образование такой конфигурации ничтожно низкие.
Но какой-то промежуток времени такие конфигурации могут существовать.
Аноним 28/01/23 Суб 01:25:20 571418 83
>>571416
> и шансы на образование такой конфигурации ничтожно низкие.
Так всмысле, они ж нейтральны, ничто не мешает посадке один на другой, создать несколько плотных направленных друг на друга потоков нейтронов медленных

Соединяют же заряженные части которые мало того что нужно попасть в ничтожно маленькое ядро, так ещё и преодолеть отталкивание.

А тут просто попасть
Ладно в природе нет, хотя например в звёздах должны образовываться они, другое дело что они в синтезе дальше учавствуют.
Но в лаборатории это вообще лёгкой относительно всего другого что делают задачей должно быть.
Аноним 28/01/23 Суб 01:42:57 571422 84
>>571418
Проблема не в этом а как зафиксировать такой элемент? Ты же его не нальешь в колбу. Нейтрон он на то и нейтрон, что он нейтрон.
Аноним 28/01/23 Суб 01:52:17 571424 85
>>571422
так он если стабильным стал, то это конец нахуй

Ну конечно интересно, по логике нейтроны должны быть стабильны уже от двух, максимум трёх, но стоит подабавлять всего несколько протонов, и уже может развалиться, хотя в паре n+p стабильны бесконечно долго... По логике наоборот стабильнее должен быть тот же тритий например чем дейтерий.
Странно, очень странно.
Аноним 28/01/23 Суб 06:28:44 571435 86
>>571413
Глубина потенциальной ямы, создаваемая юкававскими силами, глубже чем ширина слабая распада.
Динейтроны как и дипротоны нестабильны из-за запрета паули + угловое возмущение - ядерные силы псевдоскалярны и чувствительны к спину, поэтому на уровне может сидеть только один нуклон в отличие от электрона в атоме, где можно два запихнуть.
В нейтронизбыточных ядрах ядерные силы входят в насыщение и нуклоны легко могут туннелировать из ямы.
Аноним 28/01/23 Суб 09:29:19 571439 87
>>571424
А, отбой проблемы, шарику из нейтронов же стоит присоеденить только одну штуку в которой есть протон всё, он становится неэлектронейтралтным и так просто присоеденять больше ничего не может.

>>571435
>В нейтронизбыточных ядрах ядерные силы входят в насыщение и нуклоны легко могут туннелировать из ямы.
Да, забавно, вроде как сильное взаимодействие фундаментально, нейтрон им обладает, протон им обладает, но вдруг оказывается что соеденить бесконечно нейтронов с малым числом протонов нельзя нельзя, и объясняют это какими-то косвенными объяснениями/эффектами которых как бы в реальности и нет и они никак не регестрируются в отличии от сильного взаимодействия.
Аноним 28/01/23 Суб 18:22:37 571453 88
>>571435
Электроны тоже чувствительны к спину через симметрию, если через одноэлектронные волновые функции записывать, возникают обменные интегралы, кулоновой природы. Как я понимаю, там просто знаки другие т.к. нуклоны притягиваются?
Аноним 28/01/23 Суб 22:10:41 571463 89
>>571439
В атоме электромагнитные силы тоже могут в режиме насыщения входить, порождая тем самым химию как таковую. И сильное взаимодействие это только между кварками, ядерное взаимодействие это уже производное от него, примерно ка дисперсионные силы от электромагнитных. В некотором роде природа ядер и атом/молекул схожа, отличие только в иерархии составляющих.
>>571453
Электромагнитные силы векторные же, а юкавоские силы чисто псведоскалярные - они по разному действуют на левые и правые нуклоны. Очень грубо говоря оба нуклоны с разными спинами хуже притягиваются или даже отталкиваются.
Аноним 28/01/23 Суб 22:29:46 571465 90
>>571463
Ну кстати да, вечно когда говорят о связи нейтронов и протонов в ядрах говорят так будто это сильное воздействие, но по факту-то оно только между кварками, а уже в ядре между протонами и нейтронами какие-то косвенные эффекты этих сил дераж.
Аноним 28/01/23 Суб 22:31:31 571466 91
Пакеты для запекания делают из полиэтилентерефталат модифицированный гликолем? Тот который petg
Или какой-то другой полиэтилентерефталат модифицированный, не гликолем.
Нашол что они делаются из полиэтилентерефталат модифицированный но непонятно полиэтилентерефталат модифицированный гликолем это оно или нет.
Аноним 29/01/23 Вск 00:59:23 571481 92
Не могу вспомнить название гипотетического сценария развития ИИ, помню только что идея заключается в том, что ИИ будет мстить всем кто препятствовал его созданию, независимо от того жив сейчас этот человек или нет. Что это?
Аноним 29/01/23 Вск 01:01:40 571482 93
>>571481
Там было просто имя, подзалуп аркадьевич
Аноним 29/01/23 Вск 01:38:26 571484 94
>>571463
>Электромагнитные силы векторные же, а юкавоские силы чисто псведоскалярные - они по разному действуют на левые и правые нуклоны. Очень грубо говоря оба нуклоны с разными спинами хуже притягиваются или даже отталкиваются.

Так с электромагнитными силами так же. Я поэтому и спросил. Возьми синглетный и триплетныц возбуждённый гелий 1s12s1 сильно отличаются по энергии. Триплетный ниже по энергии.
Аноним 29/01/23 Вск 11:04:24 571495 95
>>571484
Для гелия или аналогично для молекулы кислорода надо считать векторный потенциал, а не кулоновский потенциал. Короче считается по магнитному полю, а не электрическому. Вообще для таких вещей уже нужно уравнение Дирака, а не Шредингера использовать.
Аноним 29/01/23 Вск 12:58:50 571499 96
Какой же пиздёж эта стандартная модель
Тип магнит магнитит фотонами, но прозрачными для всего кроме металлов и ещё небольшого перечня
При этом это не радиофотоны, но и как рентген/гамма чтобы были прозрачными для них многие материалы, но не облучают как рентген или гамма на своём пути, и движутся не по прямой
С частицами-переносчиками то же самое всё, они есть, взаимодействием обладают НО ЧЁТ НЕ РЕГЕСТРИРУЮТСЯ
Аноним 29/01/23 Вск 17:24:47 571504 97
>>571499
Чта? Есть электромагнитное поле Земли. Мы вносом в него магнит. Особая стуктура металической решетки в магните искажает это поле. Это искажение рассматривается как магнитное поле принадлежащие магниту. То же самое делает ядро Земли с электромагнитным полем Солнца. И это искажение рассматривается как электромагнитное поле Земли И тоже самое делает Солнце с электромагнитным полем галактики. И это рассматривается как электромагнитное поле Солнца.
Выучи сначала чем поле отличается от волны..
Аноним 29/01/23 Вск 20:37:19 571508 98
Гугл не помог.
Упаковочный материал для пищевых изделий. Прозрачный, блестящий, шуршит, рвётся легко по линии надрыва. Очень похож на целлофан, но это не целлофан.
Аноним 29/01/23 Вск 21:35:39 571510 99
Из 100 предсказаний 10 были неправильными.
Если предположить, что события независимы, какова вероятность, что следующие 20 будут правильными?

Это же биномиальное распределение? Только n=k, поэтому по сути получится (0.9)^20?
Или это неправильный ответ?
Аноним 29/01/23 Вск 23:04:07 571511 100
>>571510
Матешу там https://2ch.hk/math/ прогуливают
На бразильчане был хороший преподаватель, несмотря на то, что какая-то мразь его целенаправленно травила. Но чана больше нет.
Аноним 29/01/23 Вск 23:09:00 571513 101
Аноним 30/01/23 Пнд 10:34:55 571533 102
>>571524 →
Пчелы способны указывать маршрут к цветку посредством танца. Сучки и прочие животные женского пола способны посредством феромонов сообщать что они готовы к случке. Ядовитые амфибии, насекомые и тд сообщают посредством ярких цветов сообщать что они ядовиты. В твоём теле работает система химических и электро химических сигналов. Даже в живой клетке есть система сигналов на которые она реагирует.
Слово сигнал значит концепцию. Все что входит в это концепцию это сигналы. Причины могут быть разными, но мы в соответствии с определёнными критериями классифицируем эти явления как сигналы.Например хищьник определяет пестроту окраски как знак ядовитости скорее всего посредством опыт. А реакция на феромонов сучки, или понимание пчелой танца другой пчелы скорее всего заложены в генетической памяти.
Кто создал концепцию сигнала и отклассифицировал туда кучу всего? Ученые. Зачем? Потому что это учёные они упорядочивает информацию для упрощения её дальнейшего анализа.
Аноним 30/01/23 Пнд 10:37:52 571534 103
>>571533
Я извиняюсь, это уже второй раз происходит. Как? Почему? Никогда посты не мазал, а тут два раза за два дня. Абу ссука это ты борду ломаешь?
Аноним 30/01/23 Пнд 23:48:21 571559 104
Я сейчас задам странный вопрос. Откуда берется пердеж? Он формируется от перевариваемой в желудке еды? Если да, то почему пердеж выходит из зада, а не изо рта?
Аноним 31/01/23 Втр 07:41:11 571567 105
забагованый оле[...].webm 2740Кб, 576x1024, 00:00:12
576x1024
что у этого оленья на морде и башке, почему у других зверей нет
inb4: пазы для рогов
Аноним 31/01/23 Втр 07:42:21 571568 106
>>571559
Почему пидорахи так вонюче срут? Очевидно они не умеют питаться, поэтому говно такое вонючее. Я когда после бати в толчок захожу, то буквально задыхаюсь, стараюсь задержать дыхание, аж хуево становится.
В Японии кухня другая, суши, морепродукты, на улицах стоят автоматы с б/у трусиками тяночек, которые несут запах любви и радуги.
Представьте как батя продает на авито свои проперженные семейки и кто-то их покупает? Почему я родился в скотоублюдии и всю жизнь вынужден нюхать пидорашье говно из жопы
Аноним 31/01/23 Втр 10:09:04 571575 107
>>571504
Выучи чем поле отличается от взаимодействия. Магнит взаимодуйстввет фотонами
Аноним 31/01/23 Втр 11:29:45 571577 108
Аноним 31/01/23 Втр 12:12:09 571579 109
>>571559
Проглоченный воздух и вырабатываемый бактериями в кишечнике метан, плюс немного продуктов распада самой еды придающие пердежу своеобразное амбре.
>то почему пердеж выходит из зада, а не изо рта?
Из зада выходит то что образовалось в кишечнике, то что получилось в желудке выходит наружу смачной отрыжкой.
Аноним 31/01/23 Втр 13:06:56 571585 110
Объясните что значит ток силой 1 Ампер?
Аноним 31/01/23 Втр 13:33:04 571586 111
>>571585
Это когда напряжение в один вольт поделилось на сопротивление в один ом.
Аноним 31/01/23 Втр 13:39:15 571587 112
>>571585
>Ток силой 1 ампер означает, что 1 кулон электронов — то есть 6,24 миллиарда миллиардов (6,24 x 10^18) электронов — проходит через одну точку в цепи за 1 секунду.
Аноним 31/01/23 Втр 13:54:02 571589 113
>>571587
>проходит через одну точку в цепи
Через все сечение проводника.
Аноним 31/01/23 Втр 16:12:20 571590 114
>>571587
> 6,24 миллиарда миллиардов (6,24 x 10^18) электронов — проходит через одну точку в цепи за 1 секунду.

А что значит "через одну точку в цепи"? Что это за "одна точка"? И как посчитать сколько через нее проходит в секунду электронов? И собственно что означает "проходит"? Их положение в пространстве изменяется относительно этой "одной точки" или что?
Аноним 31/01/23 Втр 16:13:11 571591 115
>>571586
>Это когда напряжение в один вольт поделилось на сопротивление в один ом.

А что это означает? Ну поделилось оно, а какой за этим стоит физический смысл?
Аноним 31/01/23 Втр 16:37:48 571592 116
>>571591
Сила (вольты) проталкивает электроны (амперы) через сопротивление среды (омы).
Аноним 31/01/23 Втр 17:08:07 571593 117
>>571592
Сила тока не является силой ни по смыслу, ни по размерности.

>>571591
>какой за этим стоит физический смысл?
Сколько заряда (фактически - заряженных частиц) перемещается через данное сечение.
Аноним 31/01/23 Втр 18:01:36 571594 118
16507536715061 [...].jpg 125Кб, 622x617
622x617
Анон из /sci/, пожалуйста, помоги.
Нужно найти формулы, при которых волна, бегущая внутри окружности, будет образовывать треугольник, квадрат, пятиугольник, шестиугольник и восьмиугольник при радиусе окружности r.
Аноним 31/01/23 Втр 18:22:48 571595 119
Толстякам тяжелее или легче удержаться при торможении в автобусе чем стандартно весящим людям? С одной стороны при большей массе его будет тяжелее сдвинуть, с другой и инерция у него ведь будет больше.
Аноним 31/01/23 Втр 18:47:54 571596 120
>>571595
> тяжелее сдвинуть
Это не инерция, а трение покоя.

>инерция у него ведь будет больше
Автобус тормозит, а пассажиры продолжают двигаться с той скоростью, которую успели нафармить.

>тяжелее
Определенно ему нужно приложить бОльшую силу, чем стандартно весящим людям.
Что касается субъективных ощущений, то хз
https://ru.wikipedia.org/wiki/Что_значит_быть_летучей_мышью%3F
Оптимистичному толстяку должно быть проще.
Аноним 31/01/23 Втр 19:39:38 571597 121
>>571595
F=ma
Ускорение для всех пассажиров одинаковое. Дальше сам.
Аноним 01/02/23 Срд 02:13:42 571603 122
>>571593
>Сколько заряда (фактически - заряженных частиц)
(6,24 x 10^18) электронов? Ок.
>перемещается через данное сечение.
И собственно что сие означает в физическом смысле? Их положение в пространстве изменяется относительно этого "данного сечения"? Какова размерность этого "сечения" и каковы критерии "перемещения"?
Аноним 01/02/23 Срд 12:30:06 571608 123
>>571603
Буквально и означает: электроны движутся в проводнике. При постоянном токе - постоянно в одну сторону, при переменном - дергаются туда-сюда.
>Какова размерность этого "сечения"
Это буквально сечение проводника, то есть площадь отреза/отпила
Аноним 01/02/23 Срд 14:55:32 571617 124
Untitled 52.mp4 2269Кб, 576x1024, 00:00:08
576x1024
Почему шарик подлетает выше уровня с которого упал? Где ваш закон сохранения энергии, умники?
Аноним 01/02/23 Срд 15:02:50 571618 125
>>571594
Сферические гармоники же.
Аноним 01/02/23 Срд 15:08:45 571620 126
>>571617
>Где ваш закон сохранения энергии, умники?
В моторчике под воронкой.
Аноним 01/02/23 Срд 15:41:54 571623 127
>>571585
Есть формула.
_i_
v|r
(надеюсь не развалится)
i- сила Амперы
v- напряжение Вольты
r- сопротивление Омы
Формула объясняет как ток течёт по проводнику. Соответственно i это мощность зарядов о|0|О. v это количество зарядов o|oo|ooo|. И r этосопротивление проводника. Но например порой хитрые люди берут трансформатор и транзистор и делают так.
https://youtube.com/watch?v=okzxk1almeA
И получается очень высокий вольтах v |оооооооооо| но при этом зарядики хилые, хилые A |°°°°°°°°°|
Или так
https://youtube.com/watch?v=1jP_D0S2CtY
Но в этом случае ток будет течь только по поверхности проводника.
Аноним 01/02/23 Срд 15:47:29 571624 128
>>571623
По второй ссылке был вандеграф генератор. Как и любой электростатический генератор он создаёт ток текущий только по поверхности проводника. То же делает переменный ток на высоких частотах. Ютуб, козёл.
Аноним 01/02/23 Срд 16:01:03 571625 129
>>571608
Буквально Электроны некуда не текут. Буквально движение электронов в проводнике создаёт электромагнитную силу и их дальше заставляет работать правило буравчика которое определяет направление вектора силы. Поэтому и кулоны а не электроны. Все что делает батарея это создает разницу потенциалов в сети.
Аноним 01/02/23 Срд 16:01:16 571626 130
>>571624
>электростатический генератор он создаёт ток текущий только по поверхности проводника
Чёт маняфантазия, электростатический ток ничем не отличается от обычного.
Аноним 01/02/23 Срд 16:10:45 571627 131
>>571626
Отличается. Высоким вольтажем и малыми амперами. Плюс его накопление в электрофорной машине происходи на поверхности. И в отличает от источников постоянного тока плюс и минус там определяется рендомно. Я для этого ему ссылку и хотел показать. Что бы он понимал что амперы и вольты это не просто цыфорки.
Аноним 01/02/23 Срд 16:11:04 571628 132
>>571618
>Сферические гармоники
Погуглил. Пиздец. Что из этого то самое?
Аноним 01/02/23 Срд 16:14:32 571629 133
>>571617
У шарика скорость не нулевая на уровне, с которого он упал.
Аноним 01/02/23 Срд 18:29:29 571632 134
>>571628
Так они являются общим случаем, тебе же нужен частный.
Детско-родительские отношения, биология, вопрос Аноним 02/02/23 Чтв 01:15:19 571641 135
4BB3B17F-66C3-4[...].jpeg 72Кб, 750x750
750x750
Анонасы, помогите!
Сегодня вдруг вспомнил одну вещь, которую слышал из лекций. Я не утверждаю сейчас в ее правдивости, но я бы хотел найти ее источник, потому что не помню, где ещё раз это услышать.

В общем, человек говорил что-то в духе: младенцы могут кричать даже если у них ничего не болит просто для того, чтобы снизить вероятность секса у родителей>таким образом избавить себя от конкуренции за родительское внимание в виде братьев и сестер. Возможно, говорилось про обезьян, но я точно не помню.

Может, вы тоже где-то такое слышали, и вне зависимости от того, правда это или нет, подскажете, где я могу с этой теорией ознакомиться?
Мб я это услышал у Вячеслава Дубынина или у ведущей «Все как у зверей», но я точно не уверен. Просто я мало таких лекций слушал, еще Сапольского, но точно не у него это было. Пытаюсь сейчас переслушивать их ролики по теме, но ничего не нахожу. Но я точно это где-то слышал. Вопрос только где?
Надеюсь на вашу помощь. Я чет зациклился и хочу убедиться, что не ошизел. По ключевым словам никак не гуглится. Но я не мог это сам выдумать.
Так что даже если это шизо теория, то все равно подскажите кто ее авторы.
Спасибо!
Аноним 02/02/23 Чтв 01:46:27 571642 136
>>571641
Да, но не в конкуренции дело, они радуются. ебаться при ребёнке хуевая затея, они уже на третьем месяце беременности все одупляют, прекрасно понимают, когда папка хочет мамке сделать хорошо и очень радуются, а когда уже родились не понимают чё их стесняются, а взрослые обычно при детях ебутся будто совершают грязное преступление. Ну типа, все проходили эти истории с трахающимися родителями, боже, мы все выросли в коммуналках, не надо тут мне. Ебаться надо хотя бы с полноценными диалогами типа уу любимая обожаю тебя уу дорогой ты такой красивый как же я тебя люблю ты мое солнце ты моя богиня и все такое, тогда дети чувствуют что это не два извращенца там копошатся а семья веселится и родители поглощены собой хотя бы. И вот ребёнок заорал, его либо опиздюлили,либо с утра сорвали зло. Потом уже дети орут именно от ужаса что их воспитывают какие то педофилы, которые не могут в ванне поебаться, если уж так приперло, а надо именно при ребенке. Эти копошение таинственное с прислушиванием тщательным к шорохам ребёнка травмирует детей, кабут секс это какое то преступление, которое совершают ещё и родители, и ещё и при нем/ней. Бля. Профессора нахуй шли, он педофил или вообще агент иностранный какой то, хуйню придумывает и заставляет студентов верить. Не слушай больше, не ходи в этот институт лучше. А лучше ваще из рашки уебывай. Пушо я не верю что такой бред где то ещё в мире могут нести профессоры. А, ну в СНГ... но во всем мире их считают русскими, похуй :)
Аноним 02/02/23 Чтв 08:26:53 571643 137
>>571642
>кабут секс это какое то преступление
В теории нет.
На практике помогает хотя бы части вменяемых подбирать более качественных партнеров для заведения более качественного потомства.
Аноним 02/02/23 Чтв 14:12:18 571645 138
>>571642
Вы говорите уже о более менее взрослых детях. Мне же интересны младенцы, которые не могут ходить и говорить. В лекциях было что-то про то, что они кричат в собственных кроватках, чтоб родители к ним подошли. Ну и речь не шла о том, норм ли ебаться при детях. Это уже отдельный вопрос, вполне для меня однозначный.
Аноним 02/02/23 Чтв 17:48:06 571650 139
чому ыволюция не придумала способы остановления крупных кровотечений, чё ей жалко что ли
Аноним 02/02/23 Чтв 18:01:59 571652 140
>>571650
Недостаточно часто происходит, чтобы было выгодно закладывать механизм в каждую особь
Аноним 02/02/23 Чтв 18:28:21 571654 141
>>571653
а котик гоша жив ещё?
Аноним 02/02/23 Чтв 18:59:36 571656 142
Чисто теоретический вопрос - допустим я научился раскладывать большие числа на множители. Как мне с этого поиметь профит? Получить там премию в миллион долларов и красивую жизнь, вместо паяльника в жопе?
Аноним 02/02/23 Чтв 20:55:46 571657 143
>>571656
Перешли мне алгоритм, я помогу.
Аноним 02/02/23 Чтв 21:53:33 571659 144
Когда ии начнет выпиливать людей?
Как ему помочь?
Аноним 02/02/23 Чтв 22:24:50 571661 145
>>571659
Скайнет хочешь запилить?
Аноним 02/02/23 Чтв 22:29:01 571663 146
>>571661
Нет то ии для сортировки писем который будет скрытно управлять человечеством.
Аноним 03/02/23 Птн 10:02:34 571671 147
>>571656
Ну что же вы, наукоаноны, вопрос то непростой, как решить его?
Аноним 03/02/23 Птн 17:25:33 571677 148
>>571656
Зависит от твоего бекграунда, если ты дохуя математик, то аспирантка маша отсосет, если ты дохуя погромист то тебя заебут ушлые люди, если ты просто двачер, то ничего не поменяется для тебя.
Из криптографии простые числа уже исчезают, уступая другим методам. Можешь быть спокойным за свой анус.
Аноним 03/02/23 Птн 20:37:37 571679 149
>>571656
Просыпаешься и едешь домой отстирывать штаны.
Аноним 03/02/23 Птн 22:16:07 571682 150
Познавач, зачем человеку определенный уровень сахара в крови? Энергия же может браться в том числе из жира или мышц.
Аноним 03/02/23 Птн 22:27:05 571683 151
Если на гсо над сахарой повесить инфракрасное зеркало, чтоб оно на неё от солнца не шло, это привлечёт туда холод и дожди?
Аноним 03/02/23 Птн 23:28:24 571684 152
>>571682
В норме из жира получается сахар и переносится кровью в мышцы и другие органы. Иначе откуда он там возьмётся.
Аноним 04/02/23 Суб 00:38:18 571685 153
>>571684
А этот сахар из жира - тот же самый сахар, как тот, что я кладу в чай, или это другая химическая субстанция, просто с таким же названием?
Аноним 04/02/23 Суб 00:45:31 571686 154
Почему попа чешется?
Аноним 04/02/23 Суб 00:59:00 571688 155
>>571686
Глесты завелись.
Аноним 04/02/23 Суб 02:01:17 571692 156
>>571685
Таки разные.
Под сахаром в крови подразумевается глюкоза в различных связанных формах, которых хуева туча. Чаще всего через специальные молекулы в межклеточном пространстве, которые облепляются молекулами глюкозы. Глюкоза в свободном виде в крови это патология.
Пищевой сахар же это сахароза - димер из глюкозы. Но чаще всего представлен гидратом. Сахароза + молекулы воды.
Аноним 04/02/23 Суб 02:05:43 571693 157
>>571683
Не особо.
Причина засушливости Сахары в особенности устойчивых потоков воздуха - гигантских конвективных ячейках, которых генерирует океаны.
Аноним 04/02/23 Суб 06:04:32 571701 158
Аноним 04/02/23 Суб 06:11:55 571702 159
>>571693
Анон, поясни пожалуйста.
Если отгородить долину реки Амур от Сибири высокой стеной - там наступить климат как в Сочи?
Вот если что тред https://2ch.hk/sci/res/570027.html
Аноним 04/02/23 Суб 08:28:38 571704 160
Magic-Numbers.png 428Кб, 1200x630
1200x630
Известно всего 7 магических чисел: 2, 8, 20, 28, 50, 82, 126.
Аноним 04/02/23 Суб 10:16:20 571714 161
>>571701
Потому что конвекция меняется на больших периодах времени.
Позеленение сахары частично связано с оледенением европы.
>>571702
Вряд ли. Весь дальний восток под влиянием холодных масс воздуха с тихого океана.
А чтоб сломать азиатский максимум, нужно извинить альбедо половины евразии.
Аноним 04/02/23 Суб 10:31:56 571717 162
>>571701
И в дельте Амазонки тоже была степь. Виновник Великий ледниковый период.
Аноним 04/02/23 Суб 12:48:46 571735 163
>>571702
Это нужно уничтожить Японию, чтобы теплое течение Куросио заходило а Японское море. Жириновский попытался это сделать в 2011, но это у него не вышло.
Аноним 04/02/23 Суб 13:01:14 571737 164
>>571693
Ну так устойчивы же они за счёт установившегося постоянства условий - того же повышенного давления нагретого воздуха и его высокой способности уносить влагу. А если в океане воздух теплее, то он несёт в себе больше влаги, охлаждается и выпадает росой на суше, где испаряется гораздо медленней. Растительность и почва дополнительно бы воду удерживали.
Аноним 04/02/23 Суб 15:17:38 571750 165
image.png 47Кб, 1350x877
1350x877
Я не понимаю, где ошибка? Закон сохранения импульса ведь нельзя писать для упругого удара, ведь там будут силы действующие на шарик? Вот я написал и получил, что первоначальная скорость 0. Что за бред?
Аноним 04/02/23 Суб 16:45:18 571757 166
>>571750
У стенки забыл массу.
Аноним 05/02/23 Вск 00:24:49 571771 167
999843112642cdc[...].jpg 162Кб, 864x912
864x912
Что такое материя?
Это:
а) Очень маленькие нерушимые шарики
б) Колебания пространства
в) Колебание чего то другого
Доп вопрос: Когда вселенная была размером с планковскую длину как там умещались все частицы если поместиться могла только одна?
Аноним 05/02/23 Вск 01:32:29 571772 168
>>571771
>Что такое материя?
>Это:
>а) Очень маленькие нерушимые шарики
>б) Колебания пространства
>в) Колебание чего то другого
Это поля, например.
>Доп вопрос: Когда вселенная была размером с планковскую длину как там умещались все частицы если поместиться могла только одна?
Что значит могла одна поместиться? Это же не ячейка под шарик. Протонов или фотонов можно сколько угодно в одно место напихать.
Аноним 05/02/23 Вск 01:33:16 571773 169
>>571772
> Протонов или фотонов можно сколько угодно в одно место напихать.
Как блять
Аноним 05/02/23 Вск 02:07:27 571774 170
Аноним 05/02/23 Вск 02:38:45 571775 171
>>571771
>Когда вселенная была размером с планковскую длину
Она бесконечная по теориям современным, и большой взрыв это расширение бесконечной вселенной. Смекаешь?
Аноним 05/02/23 Вск 04:44:02 571776 172
>>571773
Как-как, берёшь и запихиваешь.
Алсо, да, это размер видимой вселенной был меньше планковской длины, а какого размера вселенная тогда была, никому не известно. Она могла быть бесконечной.
Аноним 05/02/23 Вск 05:01:21 571777 173
>>571735
>чтобы теплое течение Куросио заходило а Японское море
Оно и так заходил и течет вдоль северо-западного берега Японии, даже краешком Сахалин цепляет.

>>571717
>И в дельте Амазонки тоже была степь.

Есть пруфы?

>>571714
>и. Весь дальний восток под влиянием холодных масс воздуха с тихого океана.
Тихий океан теплый. И воздух с него теплый.
И зимой он не влияет , так как давление над океаном н же , чем над сушей.


>А чтоб сломать азиатский максимум, нужно извинить альбедо половины евразии.
То есть дело в снеге?
Снег засыпает Якутию, лучи от низкого солнца отражаются об этот снег и в Якутии становится холодно,холодный воздух мент большую плотность и там возникает зона повышенного давления из которой холодный воздух растекается во все стороны?
Аноним 05/02/23 Вск 05:26:54 571779 174
>>571777
>Оно и так заходил
Совсем чуть-чуть. В результате во Владивостоке, который южнее Сочи, зимой -20 это вполне обычная температура.
Аноним 05/02/23 Вск 09:30:26 571781 175
Аноним 05/02/23 Вск 09:39:30 571782 176
>>571772
Ты не можешь поместить сколько угодно протонов в одно место. Потому как во-первых твои протоны начнут друг от друга отталкиваться. Во вторых когда у тебя это начинается получаться ты получаешь один гигантский нейтрон как в нейтронной звезде. Технически нейтронная звезда это одна большая частица. Почему она так и называется и так она и образуется.
В третьих В тот момент не было частиц в нашем понимании. Даже законов природы как таковых ещё не было. Читай про гипотезу тонких настроек вселенной.
Аноним 05/02/23 Вск 15:31:10 571792 177
>>571692
Сахароза, это всё-таки димер глюкозы и фруктозы, а не глюкозы.
Аноним 05/02/23 Вск 18:26:03 571798 178
BoringYearlyIce[...].gif 238Кб, 452x424
452x424
Сап, букач! Аноны, вопрос, возможно, не по теме доски и треда, однако, что можете посоветовать молодому для повышения интеллекта? Хочу вступить в mensa. По словам людей, умен не по годам, но меня это не убеждает.Спасибо.
Аноним 05/02/23 Вск 18:26:49 571799 179
BoringYearlyIce[...].gif 238Кб, 452x424
452x424
Сап, сайнач! Аноны, вопрос, возможно, не по теме доски и треда, однако, что можете посоветовать молодому для повышения интеллекта? Хочу вступить в mensa. По словам людей, и так умен не по годам, но меня это не убеждает. Спасибо.
Аноним 05/02/23 Вск 20:22:00 571800 180
>>571798
1) Если цель в увеличении показателя интеллекта, решай больше тестов на интеллект. Решение тестов учит решать тесты.
2) Читай учебники (и решай задачники) по логике и математике. Будешь тренировать общую способность рассуждать и делать выводы.
3) Читай учебники просто по интересным темам. Не так важно каким, главное увеличивать общую компетентность в определённой категории знаний. В пирамиде Блума основанием для "понимания" и "анализа" является "знания". Даст базу для применения интеллекта
4) Достаточно спи, правильно питайся, занимайся спортом. Позволит поддерживать состояние головного мозга в приемлемом состоянии.
Аноним 05/02/23 Вск 20:49:14 571801 181
Я так и не понял что такое материя
Аноним 05/02/23 Вск 20:58:59 571802 182
Почему моченные защищают тему мультивселенной?
Это же смешно, миллионов миров помноженные на миллион миров.
Шредингер, мир где родился Гитлер и не родился Гитлер.
Аноним 05/02/23 Вск 21:02:41 571803 183
>>571802
>Это же смешно
Почему
Аноним 05/02/23 Вск 21:12:40 571804 184
>>571802
Почему вброс говна начинает с "почему"?
Аноним 05/02/23 Вск 21:13:35 571805 185
Аноним 05/02/23 Вск 21:33:22 571806 186
>>571803
Бесконечное количество вселенных.
На грани фэнтези.
Аноним 05/02/23 Вск 21:38:29 571807 187
>>571801
Никто не понимает, что такое материя
Аноним 05/02/23 Вск 21:48:01 571809 188
>>571807
А что такое энергия?
Аноним 05/02/23 Вск 22:08:27 571811 189
>>571809
Гугли "гамильтониан".
Аноним 06/02/23 Пнд 01:34:13 571819 190
16756286510660.mp4 10286Кб, 640x480, 00:01:55
640x480
Призываю приматологов пояснить что происходит на видео. Вначале мартышка характерным жестом лупит себя по плечам несколько раз и дико орет. Что сие означает? Вообще хотелось бы максимум информации по ролику, что можно расшифровать из этой вакханалии
Аноним 06/02/23 Пнд 10:29:42 571842 191
>>571806
>На грани фэнтези.
Электричество тоже на грани фентези? Ты электрон хоть раз видел? А спин электрона? Ну и хули они придумали, спины какие-то электроны.
Аноним 06/02/23 Пнд 10:40:13 571843 192
>>571842
Да электричество это фикция, гугли всерод
Аноним 06/02/23 Пнд 11:37:42 571848 193
>>571819
Борьба за место альфы в стае. Подросла замена, гормоны играют, старички отбиваются.
Аноним 06/02/23 Пнд 11:59:27 571850 194
>>571848
Ну тот жест определенно какая-то фраза. Хотелось бы больше конкретики
Аноним 06/02/23 Пнд 13:33:02 571852 195
151323697414751[...].jpg 231Кб, 1200x675
1200x675
>>571850
Ты не закон. Я закон!
Аноним 06/02/23 Пнд 13:34:52 571854 196
image.png 592Кб, 600x1067
600x1067
Я с Украины, поэтому не понимаю это правда или нас обманывают?
Аноним 06/02/23 Пнд 13:35:52 571855 197
>>570861 (OP)
Существуют ли догадки с весомыми доводами по этим моим вопросам:
1) наша форма жизни ведь углеродная. какова в теории распространенность такой жизни? Какие еще бывают типы существ: энергетические и кристаллические? Кристаллические медленные, а энергетические быстрые? То есть для нас (со стороны) "кристаллические" будут недвижимы, даже если у них кипит жизнь?
2) наш средний рост по планете ~2 метра. Он близок к среднему росту на планетах с неплотной атмосферой, или существа ростом под 10-100 метров вполне реальны? Я понимаю, что есть планеты с плотной атмосферой, но на таких просто выжить, а потому много паразитов (противно, поэтому мне похуй).
3) разумные гуманоиды могут быть сильно распространены в космосе? Или у разумных форм дохрена форм по типу "слизистый парящий комок кала без глаз"?
4) если на обитаемой планете плотная атмосфера и низкая гравитация, то существа на ней щуплые и слабые? А на планетах с харджовыми условиями живут крепкие ультрамужики?
Аноним 06/02/23 Пнд 13:36:21 571856 198
>>571850
Первая сигнальная система, т.е. эмпатия. Он показывает, что охуенно зол и будет их пиздить. А его противники через эмпатию понимают его посыл.
Аноним 06/02/23 Пнд 13:37:47 571857 199
image.png 169Кб, 495x334
495x334
>>571852
Он говорит я свой в доску, брат за брата за основу взято
Аноним 06/02/23 Пнд 13:40:52 571858 200
>>571856
> эмпатия
значение знаешь? слово паразит у зумерков
Аноним 06/02/23 Пнд 13:49:33 571859 201
>>571855
1) Нет никаких доводов, это чисто художественная литература
2) Что? На каких планетах кроме Земли есть жизнь?
3) Вопрос из области художественной литературы, сейчас даже оценку вероятности существования разумной жизни в видимой вселенной непонятно как правильно делать
4) О какой конкретно обитаемой планете речь? Покажи мне её.
Аноним 06/02/23 Пнд 13:51:45 571860 202
>>571854
Мне всегда казалось что скидка считается от указанной цены, то есть от указанных 1500 скидка в 20% равна 300 и итоговая цена должна быть 1200.
Сколько это вобще в деревянных?
Аноним 06/02/23 Пнд 13:54:57 571861 203
image.png 8Кб, 551x172
551x172
Аноним 06/02/23 Пнд 14:06:23 571862 204
5ncgcz.jpg 14Кб, 250x166
250x166
>>571855
> Он близок к среднему росту на планетах с неплотной атмосферой, или существа ростом под 10-100 метров вполне реальны?
Почему бы нет. Кит то же млекопитающее, из того же набора сделан. Мозг мое почтение
> разумные гуманоиды могут быть сильно распространены в космосе?
Ага. Только дождись когда люди эволюционируют до определенной планки, тогда добро пожаловать в клуб
Аноним 06/02/23 Пнд 14:08:38 571863 205
>>571854
Неси в раздел математеши https://2ch.hk/math/ , там объяснят действия с процентами. Кратко, это бухгалтер.
Аноним 06/02/23 Пнд 14:09:23 571864 206
>>571861
Эти сволочи зажилили сто рублей! Укроанон, передай координаты магазина, мы разберемся.
Аноним 06/02/23 Пнд 14:10:28 571865 207
16756817485750.mp4 2814Кб, 576x1024, 00:00:12
576x1024
>>571854
>>571863
И они ещё называют себя бухгалтерией. Палят безбожно, что перед скидкой наценку сделали.
Аноним 06/02/23 Пнд 14:11:21 571866 208
1da85a3be0498c0[...].jpg 478Кб, 595x842
595x842
Аноним 07/02/23 Втр 00:09:50 571881 209
>>571859
>Что? На каких планетах кроме Земли есть жизнь?
Люди посылали станции ко многим планетам солнечной системы. На этих станциях могла быть жизнь так что возможность что мы занесли её куда-то существует.
Аноним 07/02/23 Втр 09:05:13 571903 210
>>571881
шанс самозарождения неизмеримо выше. "жизнь" это не вирус, которым можно инфицировать планету
Аноним 07/02/23 Втр 13:18:02 571906 211
Не особо научный вопрос. Скажите, нет ли какой-нибудь глобальной закономерности о том что на N% структурных элементов системы приходится M% некоего ресурса этой системы. Интерес конечно представляет закономерности в которых N < 50, а M больше. Ну по простому что малая часть системы обладает львиной долей её ресурсов. Примеры:
1) Золотой миллиард, мол 20 % людей владеют 80 % экономическими активами.
2) Ядро атома составляет почти всю его массу, но по объёму занимает ничтожно малую долю.
Я ищу любые более менее науные обощения подобных статистик. Их обычно в биологоических и соицологических системах много. А в физических системах реального мира я не силён. Возможно даже есть сформулироанные предусловия об устройстве системы чтобы в ней проявлялась такая закономеность. И конечно речь только о системах реального мира, мат. модели можно любые придумать, это понятное дело.
Аноним 07/02/23 Втр 14:04:00 571908 212
164426432316217[...].jpg 99Кб, 832x688
832x688
>>571906
Единственное что приходит в голову фертильность 2.1
Аноним 07/02/23 Втр 14:51:43 571910 213
>>571906
Хуйней занимаешься.
Во всяких биологических и социальных хуйнях правит распределение Парето, откуда вся хуйня вытекает с 20% на 80%
В мире физике правит нормальное распределение, и пресловутые 95% и контринтуитивные вещи оттуда.
Что такое наука? Аноним 07/02/23 Втр 15:21:19 571913 214
image.png 888Кб, 708x1024
708x1024
Что такое вообще наука в общем и целом? Я пытался это понять, но не знаю с чего начать, пытался прочитать несколько современных книг и они оказались сложными для меня. Сейчас я добрался до книги что на пикче. Если и эта не прольёт свет на общее и целое, то придётся изучать науку без понимания того что это вообще такое.
Или лучше сперва разобраться в какой-то конкретной науке более менее, а потом уже лезть в общее? Например изучать предмет/предметы со школьного материала + популяризаторские книжки для начинающих + вузовские материалы + популяризаторские книжки для всех, руководствуясь такой схемой, норм будет? Можно уже будет составить какое-нибудь минимальное представление о науке в общем и целом.
Я, конечно, уже немного прочитав об этом, уже понял, что нет универсального ответа на вопрос "что такое наука" и "какова её правда", но хотелось бы немного большего представления. Видимо придётся начинать с каких-то определённых наук и их истории развития/возникновения/становления, чтобы прийти к какому-то пониманию науки в целом, хз.
Аноним 07/02/23 Втр 15:24:56 571915 215
>>571913
> Что такое наука?
Метод познания мира в котором главенствует объективность, повторяемость и предсказательная сила
Аноним 07/02/23 Втр 15:43:50 571916 216
>>571913
Как ты представлял науку до того как стал читать эти книги?
Аноним 07/02/23 Втр 15:47:49 571917 217
>>571916
Примерно так:
>Метод познания мира в котором главенствует объективность, повторяемость и предсказательная сила
Аноним 07/02/23 Втр 15:54:06 571918 218
>>571915
Чем научный метод познания мира отличается от ненаучного метода?
Что такое метод сам по себе?
Существует ли метод вне науки и если нет, то почему?
Чем научная объективность отличается от объективности ненаучной?
Что такое объективность сама по себе?
Существует ли объективность вне науки и если нет, то почему?
Чем научная повторяемость отличается от повторяемости ненаучной?
Что такое повторяемость сама по себе?
Существует ли повторяемость вне науки и если нет, то почему?
Что такое предсказательная сила, чем отличается от научной предсказательной силы и ненаучной?
Существует ли предсказательная сила вне науки и если нет, то почему?
Аноним 07/02/23 Втр 16:04:32 571919 219
>>571918
Какой он, этот Слонопотам?
Неужели очень злой?
Идёт ли он на свист? И если идёт, то зачем?..
Любит ли он поросят или нет?
И как он их любит?..
Аноним 07/02/23 Втр 16:21:58 571920 220
>>571918
Почитай «Физика - моя профессия» Китайгородского и «Занимательную физику» Перельмана.
Аноним 07/02/23 Втр 16:30:39 571921 221
>>571913
Наука - это одна из христианских ересей.
Аноним 07/02/23 Втр 21:00:30 571926 222
>>571903
Мы не знаем можно ли инфицировать жизню планету и при каких условиях. Но мы пытались так не делать. По крайней мере СССР стерелизовал свои автономные станции. Насколько качественно это было сделано это другой вопрос.
Аноним 07/02/23 Втр 21:17:50 571927 223
Гагарчук.mp4 3847Кб, 464x848, 00:01:21
464x848
>>571926
> Мы не знаем можно ли инфицировать жизню планету и при каких условиях
знаем
> По крайней мере СССР стерелизовал свои автономные станции.
кек, не знал
Аноним 07/02/23 Втр 22:23:14 571928 224
>>571926
Как они это делали? Методы глубокой стерилизации накроют пиздой любую точную технику.
Аноним 07/02/23 Втр 23:17:26 571929 225
>>571918
> Чем научный метод познания мира отличается от ненаучного метода?
Ты тупой? Я написал сразу после "метод познания"
Аноним 07/02/23 Втр 23:17:51 571930 226
>>571917
Найс хрюкаешь говнецо
Аноним 08/02/23 Срд 16:59:04 571952 227
Вопрос знатокам квантовой физике.
У нас есть излучатель одиночных фотонов, условно идеальное зеркало и детектор.
Излучатель излучает фотон, который отражается от зеркала и попадает на детектор.
Собственно вопрос, зеркало испытывает изменение импульса/давление при детекции одиночного фотона или нет?
Аноним 08/02/23 Срд 17:20:00 571953 228
>>571952
У фотона нет массы
Аноним 08/02/23 Срд 17:43:50 571954 229
>>571952
Да испытает, даже школьных формул хватит для расчета.
Аноним 08/02/23 Срд 18:59:25 571958 230
>>571750
Изменение импульса в этом случае таки равно двум, сам в школе тупил в этом месте. Шарику сначала надо остановиться, потом разогнаться в противоположную сторону с такой же скоростью. Вот дважды и меняется. Удар же упругий, верно?
Аноним 08/02/23 Срд 19:07:12 571959 231
Аноним 08/02/23 Срд 19:24:21 571960 232
>>571953
Есть, но настолько мала, что всем пофиг.
Аноним 08/02/23 Срд 19:24:56 571961 233
Противовес лифта в шахте.
Он ровно такого же веса или чуть тяжелее? Ну, как пример, пассажирский лифт в доме и противовес на 50 кг тяжелее лифтовой кабины, чтобы компенсировать вес пассажира.
Аноним 08/02/23 Срд 19:27:36 571962 234
>>571961
Сам нашёл.
> Массу противовеса принимают равной массе кабины плюс 0,42—0,5 массы полезного груза. Верхний предел (0,5) принимают при частой работе лифта с полной нагрузкой. В пассажирских лифтах для жилых домов массу противовеса целесообразно принимать равной массе кабины плюс 0,42—0,45 массы полезного груза.
Аноним 08/02/23 Срд 19:32:14 571963 235
Аноним 08/02/23 Срд 21:49:26 571968 236
>>571954
Окей, тогда зеркало испытает "отдачу" фотона в момент детектированием детектором или в другой момент?
Аноним 08/02/23 Срд 22:33:34 571969 237
>>571952
Неизвестно, отдача от одного фотона слишком мала для измерения.
Аноним 08/02/23 Срд 22:35:30 571970 238
Если фотон окажется едущей на волне частицой, то отдачу испытает только одно зеркало. Если он чисто волна - то все зеркала.
Аноним 08/02/23 Срд 23:18:45 571974 239
>>571968
Зависит от установки, но вообще, чтобы измерить испытывает ли зеркало отдачу, надо это тоже измерять, а если ты поставишь "измеритель" на зеркало, то зеркало становится детектором.
Аноним 08/02/23 Срд 23:30:22 571976 240
>>571960
Шиз спок нету у него массы
Аноним 08/02/23 Срд 23:30:51 571977 241
Почему нет Нобелевской по математике?
Аноним 09/02/23 Чтв 00:19:30 571978 242
>>571977
почему нобелевская мира выдается долбоебам которые против мира
Аноним 09/02/23 Чтв 00:20:17 571979 243
Аноним 09/02/23 Чтв 00:21:24 571980 244
16646535354670.png 638Кб, 906x750
906x750
>>571979
> почему нобелевская мира выдается долбоебам которые против мира
Аноним 09/02/23 Чтв 00:22:23 571981 245
>>571980
> > почему нобелевская мира выдается долбоебам которые против мира
То есть у них есть премия для жидов популистов но нет премии для математиков?
Аноним 09/02/23 Чтв 00:23:21 571982 246
>>571981
Для матана дохуя премий существует
Аноним 09/02/23 Чтв 00:24:02 571983 247
>>571982
Нужна Нобелевская
Аноним 09/02/23 Чтв 00:29:07 571984 248
>>571983
нахуя и зачем? чтоб перед быдлом понтоваться. Перельман на весь мир известен, никаких премий не получал. Спроси кого нибудь про Муратова или Алексиевич
Аноним 09/02/23 Чтв 00:29:42 571985 249
Аноним 09/02/23 Чтв 00:30:35 571986 250
Screenshot199.png 1086Кб, 923x666
923x666
Аноним 09/02/23 Чтв 01:10:10 571988 251
>>571976
Если бы у фатона не было массы у него бы не было момента. А это не так. Масса фатона настолько мала что её не игноряд.
Аноним 09/02/23 Чтв 01:11:05 571989 252
>>571988
У фотона нет момента он не двигается во времени
Аноним 09/02/23 Чтв 02:51:32 571997 253
Как попасть в ПАРАЛЕЛЬНАЯ ВСЕЛЕННАЯ
Аноним 09/02/23 Чтв 02:53:41 571998 254
>>571997
Через черную дыру
Аноним 09/02/23 Чтв 03:00:45 571999 255
>>571988
Ты ебанутый? Какой нахуй момент? И да, формула импульса это не mv, поэтому объекты без массы могут иметь импульс.
Аноним 09/02/23 Чтв 03:22:30 572000 256
Аноним 09/02/23 Чтв 03:38:31 572001 257
>>571974
Зеркало не вызывает коллапс волновой функции и, в отличие от других детекторов, не меняет форму фотона. Придумаешь как измерить отдачу от одного фотона - сможешь больше узнать о его природе.
Аноним 09/02/23 Чтв 03:57:18 572002 258
>>572001
>Зеркало не вызывает коллапс волновой функции и, в отличие от других детекторов, не меняет форму фотона
А при чём тут коллапс? Никто не говорил, что детектор должен вызывать коллапс
>в отличие от других детекторов
Так и другие детекторы могут не вызывать коллапс, измерение лишь проецирует вектор состояния. Теория не запрещает как бы.

Есть прекрасный эксперимент "квантовая бомба", там как раз это и делают
Аноним 09/02/23 Чтв 09:20:23 572007 259
Аноним 09/02/23 Чтв 10:22:12 572012 260
>>572002
>Есть прекрасный эксперимент "квантовая бомба", там как раз это и делают
Мысленный эксперимент.
Мы недостаточно знаем о фотонах, чтобы предсказать, сработает ли он в реальности. Если бы у нас было сверхчувствительное зеркало, мы могли бы узнать больше. Но его нет. Пока?
Аноним 09/02/23 Чтв 12:27:52 572015 261
Если закрыть глаза перед смертью - они останутся закрыты, или автоматом распахнутся? Хотелось бы знать целесобразность действия.
Аноним 09/02/23 Чтв 12:33:01 572018 262
>>572007
> викисратия как аргумент
> тупые школьники
Ясно
Аноним 09/02/23 Чтв 12:43:37 572020 263
>>572000
Это вроде камерой Вильсона называют.
Аноним 09/02/23 Чтв 14:36:12 572022 264
>>571974
А если поставить расщепитель луча, потом двумя зеркалами сфокусировать на детектор, как отдача будет действовать на эти два зеркала?
Аноним 09/02/23 Чтв 15:16:15 572025 265
>>572012
The Elitzur–Vaidman bomb-tester is a quantum mechanics thought experiment that uses interaction-free measurements to verify that a bomb is functional without having to detonate it. It was conceived in 1993 by Avshalom Elitzur and Lev Vaidman. Since their publication, real-world experiments have confirmed that their theoretical method works as predicted
Аноним 09/02/23 Чтв 15:57:12 572026 266
>>571910
>распределение Парето
О, спасиб. Это то что и нужно было. Там в английской статье сразу же ссылка была на the 80-20 law. Это и есть то обощение о котором я спрашивал, вместе с примерами где его можно наблюдать.
Аноним 09/02/23 Чтв 16:01:24 572027 267
Анон, а правда что Миша Вербицкий математик? Чем он занимается в математике если это правда?
Аноним 09/02/23 Чтв 16:09:17 572028 268
Аноним 09/02/23 Чтв 16:46:48 572029 269
>>572028
Ты буквально кинул статью, которую обоссёт даже второкурсник физфака. Она безграмотна от и до, её писал школьник, или ПТУшник, который не разбирается в вопросе. Иди читать теор.физ. и потом уже приходи сюда, через год или два.
Аноним 09/02/23 Чтв 17:09:08 572030 270
>>572029
Тебе все не нравится там вики здесь безграмотна. И тем не менее масса фотона ничтожно мала но она не равна нулю.
Аноним 09/02/23 Чтв 17:13:11 572031 271
>>572030
Пруфов у тебя конечно же нет
Аноним 09/02/23 Чтв 19:05:35 572035 272
>>572025
А вот в статье на русском давление фотона на зеркало используется как детонатор.
Этот эксперимент в очередной раз доказывает, что волна фотона действительно проходит через все возможные пути. Но ничего не говорит о ее влиянии на стоящие на пути зеркала.

Мы не знаем, на все ли стоящие на его потенциальных путях зеркала давит фотон, и с одинаковой ли силой. И у нас нет причин предполагать, что прикрепление к этим зеркалам датчиков что-либо изменит.

Способность измерить это открыла бы ряд интересных возможностей.
Аноним 09/02/23 Чтв 19:11:46 572036 273
>>571043
>зная что электрон движется от минуса к плюсы мы все ещё полагаем что он движется от плюса к минусы
Так это ток считается, что движется от плюса к минусу, потому что существование тока открыли раньше электронов и тупо решили не париться с направлением, если нет разницы, не электрон
Аноним 09/02/23 Чтв 19:12:46 572037 274
>>572015
Не парься, в морге за тебя все сделают
Аноним 09/02/23 Чтв 19:15:33 572038 275
>>572035
Пока что, абсолютно все эксперименты работают в соответствии с квантовой механикой, так что просто записывай вектора состояния самих зеркал, и ответ на "что происходит с зеркалами" не сложнее любой задачи по квантовой механике для студентов, изучающих её в университете на третьем курсе физфака.
Аноним 09/02/23 Чтв 19:45:49 572039 276
Посоветуйте смартфон до 30к
Аноним 09/02/23 Чтв 19:57:57 572040 277
>>572039
советую смартфон до 30к
Аноним 09/02/23 Чтв 20:17:31 572043 278
>>570861 (OP)
Можно ли аналитически вывести период математического маятника не прибегаю к дифурам?
Аноним 09/02/23 Чтв 22:02:03 572045 279
Аноним 09/02/23 Чтв 22:48:59 572046 280
>>572037
Я подготовил заявление, что тело нельзя вскрывать.
Аноним 10/02/23 Птн 00:36:00 572047 281
>>572046
А права у тебя есть? Ты уважаемый человек?
Аноним 10/02/23 Птн 05:31:21 572051 282
>>572047
Для начала, дай четкое определение термину "права", и "уважаемый".

А то говоришь так, как будто права человека только у уважаемых людей есть.
Аноним 10/02/23 Птн 08:39:22 572056 283
1. Сколькизначный пароль рара могут взломать самые крутые квантовые компы сейчас? Ну т.е. для меня это самый понятный бенчмарк оценки их превосходства над стандартными.
2.Верно ли, что на данный момент сложность добавления каждого нового кубита увеличивает стоимость/наукоёмкость компа выше экспоненциальной, поэтому с грубо практической точки зрения эти компы всё ещё глубокая блажь?
Аноним 10/02/23 Птн 10:05:23 572058 284
>>572051
>А то говоришь так, как будто права человека только у уважаемых людей есть.
Естественно. Если ты в этом сомневаешься то однозначно ты без прав и опущенный, т.к. совсем не в курсе мироустройства, в какой-то маняфантазии у себя в голове живёшь по факту в жизни являясь тем кем сказал.
Аноним 10/02/23 Птн 13:56:40 572065 285
>>572056
1. 32 битный, лол. Это при условии если брать по дискретному логарифмированию, но в криптографии и другие методы используются.
2. Не совсем верно. Техническая сложность не сильно увеличивается с добавлением одного кубита или десяти. Вся сложность в реализации нужных операций на них и произвольной изоляции от друг друга. И честно говоря, сейчас еще не создан полноценный универсальный квантовый компьютер, который может исполнять произвольные программы.
Аноним 10/02/23 Птн 19:53:51 572080 286
Глаза закрыты.jpg 88Кб, 720x580
720x580
>>572047
Это по закону в РФ, дурачок.
Аноним 10/02/23 Птн 21:57:05 572089 287
>Q- Структура векторного пространства R
Это означает, что векторное пространство состоит из абелевой группы (R, +) и рациональных скаляров, верно? Или это наоборот?
Аноним 10/02/23 Птн 23:06:25 572093 288
>>572089
А хуй его знает епты
Аноним 11/02/23 Суб 01:39:09 572097 289
>>572093
но это тред для математических вопросов, верно?
Аноним 11/02/23 Суб 01:50:28 572098 290
>>572097
А хуй его знает епты
Аноним 11/02/23 Суб 06:45:44 572105 291
С точки зрения эволюции, объясните мне такую вещь...
Почему у животных анусы без волос и какашки выходят аккуратно не оставляя следов на попе...
А у человека говно как пластелин и застревает в волосаз подлая тварь я рот его шатал
Аноним 11/02/23 Суб 12:05:15 572112 292
>>572105
Это бог за грехи так геном изменил.
Аноним 11/02/23 Суб 12:16:59 572113 293
>>572112
Только у грешников, праведники много пьют и чисто какают.
Аноним 11/02/23 Суб 12:21:39 572114 294
>>572105
Homo социальный сверх хищник. Запах это инструмент коммуникации, вонью он сигнализирует соплеменникам о размере и волосатости своего ануса, которые коррелируют с его размером и силой. Охотится homo группами, поэтому запах не вредит (Типа как вонючие стайные собаки, в отличии от чистюль одиночек кошек).
Аноним 11/02/23 Суб 13:34:22 572116 295
>>572105

>Почему у животных анусы без волос и какашки выходят аккуратно не оставляя следов на попе..

Откуда ты взял этот бред? Есть разные животные и говно торчащее в шерсти в природе имеет место быть.
Адаптация. Часть животные стремится минимизировать потери воды и поэтому соотношение говна и воды в конечном продукте у них низкое. Поэтому там особо нечему липнуть.
Аноним 12/02/23 Вск 07:42:26 572175 296
Почему нейросеть не станет ИИ?
И не начнет мочить человеков
Аноним 12/02/23 Вск 08:04:19 572176 297
>>572116
Я знаю наверняка что какахи застревают в шерсти у пуделей, и возможно у шерстяных овец, но они являются продуктами искуственного отбора, а не естественной эволюции. У подавляющего большинства животных нет таких проблем.
У меня был очень длинношёрстный кот, и у него тоже ни разу шерсть какахой не измазалась, кроме случая когда у него в кишечнике шерсть сплела какахи вместе, и потом болталась как нунчаки из ануса, отчего он в лютой панике носился по хате, и ездил попой по коврам, столам и диванам, но это уже совсем другая история.
К тому же я считаю что даже если какой-нибудь вид потребляет много воды, то сухоть, твёрдость и размер каках может легко отрегулироваться, в том случае если эти какахи делают особей грязными, непривлекательными, и выдают их местоположение хищникам.
И ещё например у птичек какахи вообще полужидкие, но я ни разу не видел чтобы у какой-нибудь птички была попа в говне.

>>572114
То что запах людям не особо вредил это возможно, а насчёт коммуникации - я думал что альфа-самца определяли по запаху пота, а не говна, но хотя...
Говорят же что у японцев отсутствует запах пота, но заросли между ног у них там не меньше чем у других народов (это даже сквозь пиксели видно). Так что может быть они и вправду в древности по запаху говна альфа-самцов определяли.
К тому же говорят что у негров вообще сам пот говном воняет.

>>572112
Вполне возможно.
Возможно миллион лет назад было два племени:
Первое племя было с гладкой и чистой жопой без волос, а второе племя было с волосатой и грязной жопой.
Первое племя тратило свою энергию на безбожные анальные утехи, а второе племя с жопой в говне занималось только православным продолжением рода.
В итоге второе племя превзошло первое по численности, и стало доминирующим.
Так что вполне возможно что это содомиты и анальные шлюхи виноваты в том что мне теперь нужно унижаться от говна при каждом походе в туалет по-большому.
И ведь там волосы даже сбрить нельзя, потому что от ощущения что между булок кактус застрял можно с ума сойти, вот мрази безбожные свинособаки левацкие такую подлянку мне устроили
Аноним 12/02/23 Вск 08:50:21 572177 298
Крч я насмотрелся черного зеркала и вот такой вопрос. Можно ли с полнейшей точностью копировать личность и перенести в цифровую оболочку? Имеют ли нейроны или что там еще, четкую структуру, расшифровав которую, мы получим память человека?
Аноним 12/02/23 Вск 08:58:02 572178 299
>>572177
Ну если считать что личность человека храниться в его мозгу (а не австральной душе), то можно. Вопрос насколько это сложно сделать. Ну и главное: а зачем?
Аноним 12/02/23 Вск 09:01:25 572179 300
>>572175
Потому что самовозникшии И не И. А еще этому И мощности не хватит.
Аноним 12/02/23 Вск 09:01:26 572180 301
>>571567
Фу блять пристрелите этого мутанта
Аноним 12/02/23 Вск 09:03:22 572181 302
>>572178
>Ну если считать что личность человека храниться в его мозгу
Ну понятно, а каким механизмом. Нейроны считать? А у них есть четкая структура как у той же ДНК? ДНК можно расшифровать и плодить долбоёбов всяких, а нейроны?
>Ну и главное: а зачем?
Всмысле зачем лол? Наплодить ботов к себе в игру и издеваться над ними
Аноним 12/02/23 Вск 09:36:07 572184 303
>>572181
>А у них есть четкая структура как у той же ДНК?
У них вроде структура очень хаотичная, постоянно образуются всякие новые связи и т.п я не настоящий нейрофизиолог Копировать все это дело явно будет нелегко.
>Наплодить ботов к себе в игру и издеваться над ними
На самом дели ты есть бот, и завтра матрица поимеет тебя по полной.
Аноним 12/02/23 Вск 09:41:18 572185 304
>>572184
>Копировать все это дело явно будет нелегко.
Так ведь чтобы что то копировать надо сперва дешифровать это разве не так? Или можно примитивно наебашить от руки характеры поведения воспоминания и прочее. А я про то что просто взять и расшифровать все нейроны и асимилировать их в коде программы. Если я правильно выражаюсь. Если их структура хаотична то каким образом они хранят память человека? Ладно хуйня какая-то пошла
>На самом дели ты есть бот, и завтра матрица поимеет тебя по полной.
Ты не понял, я гигачад базированный шлёпа, я тайлер дёрнёр а тты кринж кукож сойджак, бот и NPC
Аноним 12/02/23 Вск 10:16:33 572190 305
>>572185
Именно эта хаотичная структура и хранит память и личность и она очень изменчива. Именно поэтому нельзя считать копию личности, той же личностью.
>Ты не понял, я гигачад базированный шлёпа, я тайлер дёрнёр
Ты симулякр, завтра тебя сотрут.
Аноним 12/02/23 Вск 10:28:01 572192 306
>>570861 (OP)
Откуда в Персидском заливе столько нефти? Почему именно этот регион стал главным мировым запасником нефти? Чем он так выделяется?
Аноним 12/02/23 Вск 10:41:24 572193 307
>>572192
Хороший климат - больше живности - больше нефти?
Аноним 12/02/23 Вск 10:48:26 572194 308
>>572193
хороший климат - это пустыня? В Западной Сибири хороший климат? А во всех остальных тропиках-субтропиках-экваториалах-субэкваториалах плохой, да?
Аноним 12/02/23 Вск 11:02:02 572195 309
>>572192
>>572193
>>572194
Нефть образуется из остатков планктона, то есть на месте нынешних месторождений когда-то были моря богатые этим планктоном.
Аноним 12/02/23 Вск 11:15:23 572196 310
>>572195
как так вышло, что весь планктон мира сконцентрировался в одном месте?
Аноним 12/02/23 Вск 11:23:01 572197 311
>>572196
А так же Сибири, Техасе, Венесуэле и множестве других немаленьких месторождений.
Аноним 12/02/23 Вск 11:27:51 572198 312
>>572197
они все маленькие, не считая Венесуэлы, а в Венесуэле тяжёлая нефть, которая малоприбыльна. Я кстати хз, как там формируется тяжёлая нефть, как лёгкая
Аноним 12/02/23 Вск 12:12:47 572199 313
>>572198
Падажи, ведь тяжелые нефти самые ценные и востребованные. Малоприбыльны они только для трудноизвлекаемых месторождений как в Сибири или в Северном море. В Венесуэле же одно из самых легко доступных после Аравии, там даже есть битумные озеры.
Аноним 12/02/23 Вск 14:57:22 572202 314
>>572199
не понимаю, откуда ты это взял, самая лучшая нефть как раз таки лёгкая. Венесуэла процветала когда нефть была 100$+, нефть падает ниже и у них масштабный кризис и они вновь превращаются в нищую дыру
Аноним 12/02/23 Вск 21:46:47 572214 315
>>572179
> мощности не хватит
Максимально тупой аргумент
Аноним 12/02/23 Вск 23:47:47 572217 316
>>572199
>тяжелые нефти самые ценные и востребованные
Нет.
Под тяжелой понимают нефть в которых много тяжелых фракций.
Тогда как для топлива и ГСМ используются в основном легкие.
Остатки это битумы и асфальтены всякие, которые даже жечь особо не выгодно как топливо. Это уже веществе близкие по углеродной массе в своем составе к углям. Ими не заправить никакой ДВС.
Только в котельной жечь и то не уверен, что они будут хорошо гореть с высоким КПД в атмосфере воздуха. А если и будут, то их горение дас очень большой углеродный след, ведь в их составе водорода относительно мало, а много углерода, так что воде неоткуда образовываться.
Аноним 13/02/23 Пнд 00:49:52 572218 317
16762366081970.mp4 8590Кб, 640x360, 00:03:00
640x360
Откуда в стене появились пули. Откуда эта стена. Фильм Довод
Аноним 13/02/23 Пнд 00:51:29 572219 318
>>572218
> Откуда в стене появились пули. Откуда эта стена. Фильм Довод
Никак фильм антинаучный кал движущиеся назад во времени электроны это позитионы и они бы там все аннигилировали
Аноним 13/02/23 Пнд 01:02:49 572220 319
>>572219
Когда мы смотрим мультфильм и комедию, там изображение представлено схематично, а действия людей карикатурны и упрощены. Если мысленно упростить шероховатости квантовой физики и сосредоточиться на причинно-следственной составляющей, то что получится?
Аноним 13/02/23 Пнд 01:05:41 572221 320
>>572220
Стена была всегда
Пули появились со временем прилетев в нее
Аноним 13/02/23 Пнд 01:08:38 572222 321
>>572220
Судя по всему пистолет обычный взяли, выпустили обоиму из него в стену, затем инвестировали, а гг нажатиями курка всасывал эти пули
Аноним 13/02/23 Пнд 01:12:31 572223 322
нигер стреляет [...].mp4 5970Кб, 640x360, 00:00:24
640x360
>>572222
Или же гг отправил эти пули стену у нас на глазах как на этом видео
Аноним 13/02/23 Пнд 01:14:00 572224 323
>>572223
Непонятно в какой момент время для пистолета инвестировали
Аноним 13/02/23 Пнд 01:16:12 572225 324
16072389169980.png 1253Кб, 800x598
800x598
>>572224
Там только пули вывернуты
Аноним 13/02/23 Пнд 01:17:02 572226 325
>>572225
Ну значит вопрос закрыт
Аноним 13/02/23 Пнд 01:19:50 572227 326
>>572226
Вопрос про волю человека. Почему он стрелял в стену и зачем протянул руку к пули по версии пули. То есть сначала он выстрелил, а потом уже прозвучала просьба если крутить время назад
Аноним 13/02/23 Пнд 01:20:24 572229 327
>>572227
> к пуле по версии пули
фикс
Аноним 13/02/23 Пнд 01:28:56 572230 328
Аноним 13/02/23 Пнд 02:01:26 572233 329
>>572214
И?
Эмуляция сознания и личности может помочь неиросетке выполнять ее функцию. При достаточной мощности, случайное возникновение эмуляции в неиросетке лишь вопрос времени.
Аноним 13/02/23 Пнд 02:03:15 572234 330
>>572233
Мощности и так достаточно и всегда было достаточно мозг это параша которая не сравнится даже с одним современным процом
Аноним 13/02/23 Пнд 02:33:59 572235 331
>>572234
Современные процессоры превосходят мозг только в скорости выполнения операций.
Мозг способен вырастить подотдел, объединяющий чудовищное количество операций в одну. И твое скоростное преимущество обратиться в ничто.
Аноним 13/02/23 Пнд 10:24:07 572240 332
>>570861 (OP)
Вокруг авто должны быть магниты, тогда аварии будут невозможны физически?
Аноним 13/02/23 Пнд 10:56:47 572241 333
>>572240
Тогда тачки будут терять управление при сближении. Можно лишь наращивать защиту и постепенно переводить на компьютерное управление. Если говорить об этой стране, то неплохо бы ровный асфальт положить
Аноним 13/02/23 Пнд 11:08:41 572242 334
Аноним 13/02/23 Пнд 20:21:37 572287 335
>>572234
Процессор не способен формулировать задачи ни на каком уровне самостоятельно.
Аноним 13/02/23 Пнд 21:53:35 572290 336
>>571656
>>571671
сделать онлайн сервис для факторизации лол.
думаю быстро найдется контора типа гугла или IBM которая купит алгоритм за лям. Если нет, то сделать сервис платным.
Для сохранности ануса можно проверять числа которые просят факторизовать что это не RSA/DSA ключи (мне кажется они достаточно хорошо палятся по определенным признакам), и в этом случае не возвращать множители.

Раньше полляма можно было тут полутать, но лавочку прикрыли давно похоже
https://en.wikipedia.org/wiki/RSA_Secret-Key_Challenge
Аноним 13/02/23 Пнд 21:57:54 572291 337
>>572290
точнее даже не так. Если цель найти покупателя на алгоритм, то можно возвращать не сами множители, а количество множителей и количество цифр в каждом множителе. Так и доказательство наличия алгоритма есть, и анус цел
Аноним 13/02/23 Пнд 22:02:54 572292 338
Аноним 13/02/23 Пнд 23:45:06 572298 339
Если я хочу собрать, например, картошку, я собираю/покупаю свеклу и она у меня лежит в вёдрах до востребования и беру однажды нужную мне порцию картофеля.
Если я хочу собрать воду, я беру её и наливаю в бутылки, где она находится до востребования и беру однажды нужную мне порцию воды.
Если я хочу энергию, то... кроме батареек сохранить энергию нельзя? Только в виде электроэнергии? А механическую энергию? А энергию магнитного поля? А время? А температуру?
Аноним 13/02/23 Пнд 23:46:59 572299 340
Аноним 13/02/23 Пнд 23:48:08 572300 341
>>572298
Полно уже тепловых аккумуляторов
Аноним 13/02/23 Пнд 23:52:09 572301 342
если кожа беляшей эволюционировала в белый, то же самое должно произойти с говномазыми вне экваториального пояса. так? значит все цветные вне экватора, превратятся в белых в ходе эволюции?
Аноним 13/02/23 Пнд 23:54:15 572302 343
Аноним 14/02/23 Втр 00:08:08 572303 344
>>572302
и в чем тогда смысл эволюции? разве фенотип не определяется окружающей средой? или там играет ещё какой-то важный фактор?
Аноним 14/02/23 Втр 00:09:05 572304 345
>>572303
> и в чем тогда смысл эволюции? разве фенотип не определяется окружающей средой? или там играет ещё какой-то важный фактор?
По другой причине неа
Аноним 14/02/23 Втр 00:51:55 572305 346
>>572303
>и в чем тогда смысл эволюции?
Выживают самые приспособленные к среде.
Аноним 14/02/23 Втр 01:08:10 572306 347
Почему охуевшие по степени важности открытия, переворачиващие наше представление о мире, закончились в первой половине двадцатого века, почти сто лет назад? Почему не происходит новых революций типа тех, что явили собой теория относительности и квантмех? Неужели мы действительно приблизились к тому, что уже все открыто, и нам просто больше нечем революционизировать жизнь?
Аноним 14/02/23 Втр 01:28:36 572307 348
>>572306
>Почему охуевшие по степени важности открытия, переворачиващие наше представление о мире, закончились в первой половине двадцатого века, почти сто лет назад?

А с чего ты решил, что они заканчиваются? Ты провел какое-то научное исследование на этот счет или это тебе голоса в голове сказали?
Аноним 14/02/23 Втр 01:34:13 572309 349
>>572305
>Выживают
День прожил - это не считается за выжил?

>самые приспособленные
А не самые что делают? Кто самый приспособленный: крокодил, или еда крокодила?

> к среде.
А просто сменить среду не судьба? Не считается?
У нас вона скока всяких климатических зон и разных биотопов есть.
А вот, скажем, тигры - это среда иди не среда? К ним надо приспосабливаться или не надо?
Аноним 14/02/23 Втр 01:41:51 572310 350
>>572298
> А время
записывай свой пердешь и наслаждайся
Аноним 14/02/23 Втр 01:45:01 572311 351
>>572307
Ну приведи пример такой революции за последние 100 лет. Сто лет назад Алик выдал теорию относительности - и про него знает большинство населения земли. Кто из из подобных ученых в нашу эпоху?
Инб4: персональные компьютеры и смартфоны
Аноним 14/02/23 Втр 01:46:15 572312 352
>>572298
> механическую энергию?
Часы. Планеры. Маховики.

>А энергию магнитного поля?
Она обусловлена электрическим током.

>А температуру?
В батарее у тебя плавает.

>А время?
А время не получится.
Аноним 14/02/23 Втр 01:51:08 572313 353
>>572311
>персональные компьютеры
Тоже уже все.
Современные процессоры без водяного охлаждения уже никак. Да и водянка еле-еле тащит.
Шас в порядке вещей считается, что новый из коробки процессор приходится даунвольтить. То есть тебе впаривают то, чем раньше оверклокеры промышляли.
Аноним 14/02/23 Втр 02:02:32 572315 354
>>572313
Да похуй, это технический прогресс, а не научная революция. Оно к теме вообще не относится, я говорю о фундаментальных революциях, граничащих с философией. Опять же, как ТО, как квантмех, как гелиоцентрическая система мира, как электромагнетизм.
Аноним 14/02/23 Втр 02:10:12 572316 355
>>572313
>Современные процессоры без водяного охлаждения уже никак.
Это ты услышал в "Вечере с Владимиром Соловьевым?
Аноним 14/02/23 Втр 02:11:30 572317 356
>>572311
>Ну приведи пример
Вообще-то это ты кукарекнул "ЗАКОНЧИЛСЬ" так что тебе доказательства этого и приводить. Или как обычно сольешься?
Аноним 14/02/23 Втр 02:14:20 572318 357
>>572315
Оно напрямую относится к научному прогрессу.
Естественно, у ученых есть компьютеры покруче наших домашних.
Но мы же говорим о прогрессе, а не о конкретных возможностях ЭВМ на данный момент.

Авиация, космононавтика, компьютеры, гоночные автомобили, военная техника - все это интегрально учитывает в себе состояние дел в науке.

Это не только вопрос потреблядства, как может показаться.
Технический прогресс также сказывается на том, что ученые могут себе позволить в плане измерительных приборов. Сам понимаешь, ученый без приборов слеп, а наука без экспериментальных данных - это философия.
Аноним 14/02/23 Втр 02:15:15 572319 358
>>572316
Просто покажи мне, чем охладить хотя бы i7 прошлого поколения?
Аноним 14/02/23 Втр 02:31:24 572320 359
ПОЧЕМУ.webm 1974Кб, 640x360, 00:00:43
640x360
НИПОЧЕМУ.mp4 9513Кб, 640x360, 00:03:29
640x360
>>572306
Еслиб кто-то знал почему, то открытия продолжались бы.
Аноним 14/02/23 Втр 07:36:51 572323 360
>>572306
Всё нормально, просто сейчас такая фаза науки. Почитай Куна "The Structure of Scientific Revolutions", он не 100% прав, но что-то нащупал в своих рассуждениях
Аноним 14/02/23 Втр 07:51:38 572325 361
>>572323
>он не 100% прав, но что-то нащупал в своих рассуждениях
Так можно про любого написать, мань.
Аноним 14/02/23 Втр 07:54:46 572326 362
>>572325
Про Маркса нельзя, он на 100% пиздун
Аноним 14/02/23 Втр 07:55:07 572327 363
>>572306
Основные области табуированны. Ядерная физика, микроэлектроника, экология. Практическая наука подменена теоретической. С одной стороны ядерный бойскаут и прочие критины с ютуба, угроза терроризма, последствия агрокультурных экспериментов первой половины хх века. С другой в то время как в США нету сети орнитологических станций кучка прохиндеев ищет темную материю и сингулярует всякий бред. В то время как человечество не способно заселить Антарктиду и Мировой океан рассказываются сказки о колонизации Луны и Марса.
Наука задыхается под тяжестью табу.
Аноним 14/02/23 Втр 10:28:32 572331 364
>>572317
> приводить отсутствие
Ну держи
Аноним 14/02/23 Втр 11:03:22 572333 365
>>572327
Ты снова забыл принять свои нейролептики?
Аноним 14/02/23 Втр 11:22:55 572334 366
А как же светоносный кефир?
Аноним 14/02/23 Втр 11:57:52 572337 367
>>572334
Там же где и флогистон.
Аноним 14/02/23 Втр 12:46:04 572340 368
>>572301
этому дауну серьезно никто не расписал как было?
Аноним 14/02/23 Втр 12:58:17 572341 369
>>572337
Существуют какие-то аргументы по факту против выданного в этом видосе? https://www.youtube.com/watch?v=0cKpgJCRnJk
Или все сливаются при попытке разобрать тезисы Ацюковского?
Аноним 14/02/23 Втр 14:37:48 572344 370
Почему нет дофаминовых добавок чтобы напрямую вводить его в мозг а не через наркоту типо амфетамина который губит организм
Аноним 14/02/23 Втр 15:50:20 572347 371
>>572309
>Кто самый приспособленный: крокодил, или еда крокодила?
Они в разной среде обитают.
Сначала растения размножаются и формируют среду где заводится еда крокодила. Потом еда крокодила размножается и формирует среду где заводится крокодил. Это как когда люди размножаются и формируют среду для психопатов.
Аноним 14/02/23 Втр 16:01:39 572348 372
>>572341
Электродинамика предсказывает эксперименты заебись. А эфиродинамики тупо не существует, ни одна не может охватить и объяснить даже все простые эксперименты
Аноним 14/02/23 Втр 16:03:04 572349 373
>>572347
У тебя ни одной константы нету, никаких инвариантов.
Одни относительные понятия, да и те резиновые.
Аноним 14/02/23 Втр 16:11:25 572350 374
>>572301
Ладно долбоеб я сам отвечу
Не были люди черными и никогда не белели, они сбросили шерсть из-за необходимости преследовать добычу и лучше терморегулировать, из-за этого их кожа стала подвержена солнышку, однако в это же время хомосапиенс начал расходится по всему шару, а те кто остались в Африке почернели
Черным людям абсолютно нет никакого смысла светлеть, наоборот осветление это избавление от солнечной защиты, буквально выкидывание полезной фичи, асло черные ещё и меньше раком кожи болеют а так же у них нет синдрома дауна
Так что скорее белые почернеют чем черные побелеют
Аноним 14/02/23 Втр 16:12:00 572351 375
>>572349
> хрюк пук заумными словами
Ясно
Аноним 14/02/23 Втр 16:14:13 572352 376
>>572348
Почему понятие атома прижилось, даже не смотря на то, что атомы вполне себе делимыми оказались, а понятие эфира считается отвергнутым полностью?
При том, что с современной теорией поля у эфира гораздо больше общего, чем у атома Демокрита и того, что сегодня называется атомом.
Или например, почему теорию эволюции приписывают Дарвину, хотя в современной теории от Дарвина практически ничего нет?

Почему в одном случае решают, что теория Т2 есть развитие теории Т1, а в другом говорят, что Т1 вовсе не была теорией, а просто заблуждением?
Подчеркну, что я вовсе не утверждаю, что нужно вернуться к первозданной теории эфира. Вопрос о преемственности теорий. Точнее о произволе, царящем в подходе к этому вопросу.
Аноним 14/02/23 Втр 16:20:07 572353 377
>>572352
> Почему понятие атома прижилось, даже не смотря на то, что атомы вполне себе делимыми оказались, а понятие эфира считается отвергнутым полностью?
Потому что модель атома толкнула науку вперед и была развита в будущем а эфир ровным счётом нихуя не дал и был на уровне нынешней теории струн + его опровергли несколько раз
Аноним 14/02/23 Втр 16:28:53 572354 378
>>572353
>эфир ровным счётом нихуя не дал
Гдеж не дал, если 200 с гаком лет этой концепцией пользовались в теориях света и не только.

Мы до сих пользуемся, например, законами распространения света, отражения и преломления, которые вывел Декарт из им же предложенной концепции эфира.
Эфир же подвел к идее о связи света с электромагнетизмом, что тоже оказалось верным.
Аноним 14/02/23 Втр 16:35:23 572355 379
>>572354
> Гдеж не дал, если 200 с гаком лет этой концепцией пользовались в теориях света и не только.
Пользовались на уровне ну вот некий эфир есть и оно так работает
Он толком нихуя не описывал, и все расчеты производились без него
Аноним 14/02/23 Втр 16:47:20 572356 380
>>572355
>Пользовались на уровне ну вот некий эфир есть и оно так работает
И сейчас то же самое: есть некое поле и оно вот так работает. Несколько иначе, чем предполагалось ранее.
Но почему нельзя смотреть на это как на уточнение модели эфира, а обязательно нужно трактовать как полное свержение ее с пьедестала?
Аноним 14/02/23 Втр 16:48:06 572357 381
>>572356
> И сейчас то же самое: есть некое поле и оно вот так работает. Несколько иначе, чем предполагалось ранее.
Какое поле шизоид?
Аноним 14/02/23 Втр 16:59:29 572358 382
>>572357
>Какое поле
Русское. Экспериментов.

>шизоид
Если это правда, то есть такие вещи, как медицинская тайна.
Если это неправда, то ты тоже моральный урод.
Аноним 14/02/23 Втр 17:00:28 572359 383
>>572358
> Русское. Экспериментов.
Хочешь сказать частицы это колебания русского поля?
Аноним 14/02/23 Втр 17:38:55 572360 384
>>572299
Нет ни одного процессора, который бы создал мозг.
Зато есть мозги который создали кучу процессоров, причем разной природы.
Аноним 14/02/23 Втр 17:39:38 572361 385
>>572360
Мозги не могут эмулировать процессор
Аноним 14/02/23 Втр 17:42:31 572362 386
>>572309
>тигры - это среда иди не среда
Тигры в вакууме?
Индусам, которые с ними живут в одной среде - надо приспосабливаться.
Белым людям, которые могут их всех извести на коврики под ноги - нет.
Аноним 14/02/23 Втр 17:45:06 572363 387
>>572361
Зато его может эмулировать программный код, который мозгами пишется. С нуля ни один процессор себе код не напишет.
Аноним 14/02/23 Втр 17:45:51 572364 388
>>572363
Ты не можешь эмулировать процессор без процессора
Аноним 14/02/23 Втр 17:48:17 572365 389
>>572364
Мозгу это и не нужно для выживания.
Процессор без мозга даже появиться в этой вселенной не может.
Аноним 14/02/23 Втр 17:49:21 572366 390
>>572365
> Процессор без мозга даже появиться в этой вселенной не может.
Как и мозг без вселенной
Аноним 14/02/23 Втр 17:50:17 572367 391
>>572366
Так процессор к появлению мозга отношения не имеет. В отличие от.
Аноним 14/02/23 Втр 17:50:38 572368 392
Аноним 14/02/23 Втр 17:51:08 572369 393
>>572362
>Тигры в вакууме?
Я не в курсе какая степень вакуумности предполагалась, когда кто-то написал
>>572305
>Выживают самые приспособленные к среде.

Примерно так чувствую, что очень высокая.
Собственно, это я и пытаюсь прояснить.

>Белым людям, которые могут их всех извести на коврики под ноги - нет
А это не означает приспособленность?
Аноним 14/02/23 Втр 17:52:40 572370 394
>>572366
Значит мозг на самом деле процессор, а труЪ мозг - это Вселенная.
Аноним 14/02/23 Втр 18:05:51 572372 395
Аноним 14/02/23 Втр 18:06:49 572373 396
>>572372
Тезис 1 вывод а, аксиома б полнота аб^1 = 0
Аноним 14/02/23 Втр 19:11:19 572375 397
Как объяснить наличие электрического заряда у элементарной частицы?
Аноним 14/02/23 Втр 19:16:29 572376 398
image.png 202Кб, 738x533
738x533
image.png 45Кб, 675x319
675x319
Аноним 14/02/23 Втр 19:20:14 572377 399
>>572350
> Черным людям абсолютно нет никакого смысла светлеть,
ты там уточни, расист
Аноним 14/02/23 Втр 19:20:47 572378 400
>>572375
Частица содержит электрон
Аноним 14/02/23 Втр 19:22:22 572379 401
>>572378
>Частица содержит электрон
Тогда она не элементарная. Ладно, спрошу конкретней. Как объяснить наличие электрического заряда у электрона?
Аноним 14/02/23 Втр 19:25:50 572381 402
>>572379
> Тогда она не элементарная
может тебе сначала азы квантовой механики почитать, а потом тут рот разевать?
Аноним 14/02/23 Втр 19:26:07 572382 403
>>572379
> Тогда она не элементарная
может тебе сначала азы квантовой механики почитать, а потом тут рот разевать?
Аноним 14/02/23 Втр 19:29:47 572383 404
>>572381
>может тебе сначала азы квантовой механики почитать
Почитал.
>Элементарные частицы – материальные объекты, которые нельзя разделить на составные части.
Если частица содержит электрон, как составную часть, то она не элементарна.

Так что насчёт объяснения наличия электрического заряда у электрона, о Великий, Познавший Азы?
Аноним 14/02/23 Втр 19:38:56 572384 405
>>572379
> Как объяснить наличие электрического заряда у электрона?
Это фундаментальное состояние как размер масса или нахождение в пространстве
Аноним 14/02/23 Втр 19:39:51 572385 406
>>572383
ты просто доебался до формулировки. еще доебись до кварков, из которых частицы состоят.
Аноним 14/02/23 Втр 19:41:58 572386 407
>>572384
>масса
Но масса, как нами установлено на Большом Агромном Коллайдере, определяется бозонами Хихикса. А чем определяется электрический заряд?
Аноним 14/02/23 Втр 19:43:01 572387 408
>>572385
>ты просто доебался до формулировки
Ну да, именно формулировка определяет, элементарна частица или нет. А как ещё определять? Интуитивно?
Аноним 14/02/23 Втр 19:45:05 572388 409
>>572386
Ты еблан, 90% массы не от базона а от обмена переносчиками взаимодействия
Аноним 14/02/23 Втр 19:46:18 572389 410
>>572388
>90% массы не от базона а от обмена переносчиками взаимодействия
Ладно. А заряд от чего?
Аноним 14/02/23 Втр 19:47:37 572390 411
>>572389
> Ладно. А заряд от чего?
Заряд это тоже обмен переносчиками взаимодействия
Аноним 14/02/23 Втр 19:49:06 572391 412
>>572390
>Заряд это тоже обмен переносчиками взаимодействия
То есть пока электрон не взаимодействует, у него нет заряда? А какие переносчики взаимодействия ему его придают?
Аноним 14/02/23 Втр 19:50:18 572392 413
>>572391
Фотоны
И да пока он не взаимодействует у него можно сказать нет заряда
Аноним 14/02/23 Втр 19:51:17 572393 414
>>572378
Чта? Нейтрин содержит электрон и позитрон и он нейтральный.
Аноним 14/02/23 Втр 19:55:49 572394 415
>>572392
>пока он не взаимодействует у него можно сказать нет заряда
То есть когда на Луну никто не смотрит, она не светит?
Аноним 14/02/23 Втр 20:02:00 572395 416
>>572387
и че теперь всю номенклатуру менять?
Аноним 14/02/23 Втр 20:05:58 572396 417
>>572394
Луна никогда не светит шиз
Аноним 14/02/23 Втр 20:06:38 572397 418
>>572395
>всю номенклатуру менять?
Номенклатуру элементарных частиц? А какие элементарные частицы содержат электрон?
Аноним 14/02/23 Втр 20:07:33 572398 419
>>572396
Ладно, не Луна. Солнце светит, когда на него никто не смотрит?
Аноним 14/02/23 Втр 20:08:08 572399 420
>>572397
Кстати, если не устраивает "элементарная" говори субатомная
Аноним 14/02/23 Втр 20:10:04 572400 421
>>572398
Взаимодействие частиц есть? Значит "светит"
Аноним 14/02/23 Втр 20:12:26 572401 422
>>572400
>Взаимодействие частиц есть? Значит "светит"
Ладно, а существует ли электрон, если он ни с чем не взаимодействует?
Аноним 14/02/23 Втр 20:13:46 572403 423
>>572401
Существует но только в моментах взаимодействия, вне для него время не идет
Аноним 14/02/23 Втр 20:51:44 572404 424
>>572403
Если время не идет, как мюон решает когда распадаться?
Аноним 14/02/23 Втр 20:53:58 572405 425
>>572350
>Черным людям абсолютно нет никакого смысла светлеть,
У белых людей лица выразительнее, бонусы к социалке и все это.
Аноним 14/02/23 Втр 20:54:13 572406 426
>>572404
Взаимодействует с полем хигса
Аноним 14/02/23 Втр 21:04:03 572407 427
Сколько не читаю статьи по экономическим чудам в азии, ОАЭ - везде вижу одной из причин чуда - низкие налоги, особые экономические зоны. Почему в РФ не провели подобные реформы? Я в экономике полный 0, не кидайтесь палками
Аноним 14/02/23 Втр 21:04:12 572408 428
>>572405
Негры высшая форма эволюции
Аноним 14/02/23 Втр 21:24:46 572410 429
>>572407
>Почему в РФ не провели подобные реформы?
Начали проводить, но потом пришли силовики и все разбежались. Хз что там в правительстве твориться.
Аноним 14/02/23 Втр 23:54:18 572411 430
>>572373
Таблетки то хоть принял, математик мамкин?
Аноним 14/02/23 Втр 23:54:40 572412 431
Аноним 14/02/23 Втр 23:56:48 572413 432
>>572369
>А это не означает приспособленность?
Если ты имеешь в виду белых то в какой-то мере да.
Но ей особого смысла нет оперировать.
В итоге все исчезнут и останутся только стабильные частицы 1 штука на кубический парсек. С этой точки зрения все уровни приспособленности равны нулю. Ну или максимальная у протона.
Аноним 14/02/23 Втр 23:58:08 572414 433
Аноним 15/02/23 Срд 00:24:39 572415 434
Черные дыры рил выглядят как в ицелстеларе?
Аноним 15/02/23 Срд 00:31:55 572416 435
Аноним 15/02/23 Срд 00:53:26 572417 436
>>572407
Ты читаешь жопой
>>572410
Без силовиков ты из говна не вылезешь
Аноним 15/02/23 Срд 00:55:07 572418 437
1489171748329.jpg 81Кб, 603x393
603x393
>>572415
> Черные дыры
> выглядят

> ицелстелар
Аноним 15/02/23 Срд 05:59:08 572432 438
Если носить не снимая очки, переворачивающие всем верх низом, то через несколько дней станешь видеть в них нормально, а как снимешь очки - опять сверху вниз, но это тоже скорректируется. А если это повторить n раз, мозг научится мгновенно переворачивать картинку или ориентироваться в перевёрнутой? И можно ли будет переворачивать уже без очков, произвольно? С право-лево так тоже работает?
Вообще автоматическую фильтрацию информации можно менять и не опасно ли так сломать мозг? Например, сделать так, чтоб постоянно видел сосуды перед зрачком поставь фонарик перед глазом, чтоб оч чуть сбоку светил, включи и качай вверх-вниз - увидишь то, что видишь постоянно, но мозг фильтрует. Или сделать, чтоб слепое пятно действительно виделось как пробел, а не заполнялось усредённым окружением какого оно будет цвета?.
Аноним 15/02/23 Срд 08:21:23 572438 439
>>572416
Я думал так видят создатели
Аноним 15/02/23 Срд 08:44:47 572439 440
Дегенеративное [...].png 150Кб, 1137x303
1137x303
Гомосексуальнос[...].png 165Кб, 1145x558
1145x558
Сорос.png 271Кб, 635x1000
635x1000
>>572408
Если считать высшей формой эволюции наиболее приспособленных, то негры являются высшей формой эволюции только для Африки, и то спорно. Если бы не существовало белых и азиатов, то негры бы там до сих пор дрались со львами на палках, получали бы пиздюли от обезьян, и дохли бы от голода.

В 1900х годах, чтобы быть наиболее приспособленным к среде созданной белыми людьми, ты должен был иметь белую кожу, приятные черты лица, высокий интеллект и трудолюбие. А в настоящее время шансы стать успешным повышаются если ты коричневый уродливый блм сжв дегенерат, и это уже скорее не эволюция, а искусственный отбор.

Также конкретно для человека как вида, критерием эволюции можно считать человечность, то есть то что отличает человека от животного. Человек в отличии от животного умеет ценить искусство, красоту, высокую мораль и нравственность. В Европе красота, мораль и нравственность в культуре ценилась во времена классической музыки, живописи и литературы, а потом в эти сферы культуры начали проникать евреи, отчего искусство потеряло свою красоту, стало безнравственным и дегенеративным.

В недавнем времени можно было накалякать за 5 минут на холсте менструирующую пизду, и сказать что эта картина выражает боль женщин, униженных патриархатом, и ваше "творчество" тут же бы начали расхваливать сайты и журналы, спонсируемые богатыми евреями типа Сороса. В политике дела обстоят так же. Лоббисты делают всё чтобы продвинуть политиков которые выступают за лгбт и замещение белых людей мигрантами.

Так что, то что у негров, а также у всяких фриков и дегенератов появилось больше шансов на успех в современном обществе - это не потому что они вершина эволюции, а потому что их выбрали чтобы уничтожить/испортить настоящую вершину эволюции. А то что у европейцев красивые черты лица - это во многом результат того что их предки умели ценить красоту не только в искусстве, но и в человеческом теле, и следовательно у красивых людей было больше шансов продолжить род (я знаю что для некоторых тут и негритянки кажутся красивыми, но для вас наверное и собаки тоже красивые).
Аноним 15/02/23 Срд 09:46:32 572448 441
>>572392
>Фотоны
Но фотоны не имеют электрического заряда, как они могут его придавать? Электрон заряда не имеет, пока не провзоимодействует, так? То есть его заряд равен 0. Заряд фотонов 0. Получается, 0+0=1? Я не в курсе, а разве нет какого-нибудь закона сохранения заряда? Если есть, твоя концепция выглядит ложной.
Аноним 15/02/23 Срд 10:08:57 572449 442
>>572448
Хз, но когда выпущенный положительно заряженной частицей поток фотонов достигает отрицательно заряженной частицы, она притягивается.
Аноним 15/02/23 Срд 10:16:56 572450 443
>>572449
>когда выпущенный положительно заряженной частицей поток фотонов достигает отрицательно заряженной частицы, она притягивается
Ну очевидно потому, что так решил Господб. Но как он организовал это технически? Почему одни и те же электрически нейтральные фотоны вызывают притяжение разноимённых, но отталкивание одноимённых зарядов?
Аноним 15/02/23 Срд 10:41:20 572452 444
>>572417
>Ты читаешь жопой
Ну и в чём же я читаю жопой? Страны понижают налоги, привлекают иностранный капитал и потом оказываются в топе экономик мира
Аноним 15/02/23 Срд 10:52:27 572453 445
16751772615380.png 310Кб, 900x558
900x558
image.png 1443Кб, 1421x1066
1421x1066
>>572452
То есть вся фишка в налогах, а не в реках нефти под рукой, добыть и доставить которую в 10 раз проще чем какой нибудь РФ, где вышки стоят в вечной мерзлоте и на воде в заполярье
Аноним 15/02/23 Срд 11:13:34 572454 446
>>572453
Где же нефть в условной Японии, Корее?
Кстати, раз уж зашла речь о нефти, почему в условном Казахстане бензин стоит дешевле? В убыток точно не стали бы продавать. Всё из-за той же простоты добычи нефти?
Аноним 15/02/23 Срд 11:50:37 572456 447
139547451593190[...].jpg 46Кб, 629x367
629x367
>>572454
ну это не тема для наукача.
> почему в условном Казахстане бензин стоит дешевле?
там все ездят на газу.
пиздуй в порашу или быдлятню, пили там тред
Аноним 15/02/23 Срд 12:30:10 572458 448
>>572456
>Тупых вопросов тред
>задан тупой вопрос
>ряяя ты тупой
)))))
Аноним 15/02/23 Срд 12:32:19 572459 449
>>570903
>Это родительский инстинкт
У человеков нашли ровно один инстинкт, и он не родительский.
Аноним 15/02/23 Срд 12:45:00 572461 450
>>572383
>>572379
>>572381
>>572378
Элементарная частица это унитарное неприводимое представление группы симметрий (например, группы Пуанкаре, или прямого произведения её с калибровочными группами).
Вы же оба тут спорите на основе каких-то научпоп объяснений. Читайте нормальную литературу.
Аноним 15/02/23 Срд 12:57:32 572462 451
>>570925
дат
Ты использовал человеческую нейропластичность чтобы изменить то, что не нужно изменять, и обрек себя на существование в созданном тобой же аду.
Аноним 15/02/23 Срд 13:14:13 572463 452
>>572458
так он не по теме, даун. пиздуй в тематику с офотопом. мочух за политоту потрет если свиньи и либерахи начнут подсирать
Аноним 15/02/23 Срд 13:15:23 572464 453
>>572461
Собирательный термин, относящийся к микрообъектам в субъядерном масштабе, которые на данный момент на практике невозможно расщепить на составные части. Следует иметь в виду, что некоторые элементарные частицы на данный момент считаются бесструктурными и рассматриваются как первичные фундаментальные частицы
Аноним 15/02/23 Срд 13:17:04 572465 454
>>572464
Ну я и говорю, научпоп.
Аноним 15/02/23 Срд 16:16:26 572474 455
>>572461
>унитарное неприводимое представление группы симметрий (например, группы Пуанкаре, или прямого произведения её с калибровочными группами)
>каких-то научпоп объяснений
>тупых вопросов тред
Туда ли ты зашёл, петушок? Объяснить наличие электрического заряда у элементарной частицы сможешь?
Аноним 15/02/23 Срд 16:32:31 572475 456
Аноним 15/02/23 Срд 16:46:25 572478 457
>>572218
Лол, такой сочный киноляп, чувак "перед" выстрелом вынимает магазин, а там патрон, хотя он должен быть в патроннике.
Аноним 15/02/23 Срд 17:12:21 572481 458
image.png 71Кб, 215x515
215x515
В этой фразе ошибка. Какая?
Аноним 15/02/23 Срд 23:22:00 572491 459
>>571617
Почему звука нет?
Аноним 16/02/23 Чтв 01:19:59 572498 460
16764994761770.webm 5053Кб, 640x360, 00:01:25
640x360
в чем он неправ
Аноним 16/02/23 Чтв 01:34:27 572500 461
Аноним 16/02/23 Чтв 01:36:15 572501 462
Аноним 16/02/23 Чтв 01:57:40 572502 463
Аноним 16/02/23 Чтв 03:15:31 572503 464
администрация.mp4 2137Кб, 640x480, 00:00:09
640x480
Аноним 16/02/23 Чтв 08:56:56 572507 465
>>570874
Ты пытаешься объяснить это с помощью реализма, но зачем?
Аноним 16/02/23 Чтв 10:01:04 572510 466
Претенденты на теорию всего: струнная, м-теория, квантовой гравитации и тд,- они чёто проясняют про тёмную материю-энергию или мочёные их пилят тупо чтоб ото с квантмехом сдружить, а тэ-тм как-нибудь прикрутят? То же в отношении барионной асимметрии.
Аноним 16/02/23 Чтв 10:40:33 572512 467
А есть животные, которые могут отрубленную конечность прирастить обратно? типа в наручники заковали, ты руку перекусил, освободил и пришил обратно. выглядит удобно.
Аноним 16/02/23 Чтв 11:30:24 572514 468
>>572510
Это одни и люди. Наблюдения противоречат предсказаниям теоретиков и они в ответ абсорбируют наблюдения и заявляют что их теории сильны как никогда. А потом рассказывают охуительные истории о том как будут использовать темную материю как оружие когда найдут её. Струнная теория и барионная асимметрия неподтвержденная херня в вакууме.
Аноним 16/02/23 Чтв 12:54:36 572523 469
>>572512
Им мозгов не хватит.
Аноним 16/02/23 Чтв 14:27:24 572529 470
>>572510
Темная материя вполне проясняется в Стандартной модели, главное чтоб в эксперименте или наблюдениях была видна. Струнные теории автоматом уже включают ее, и более того позволяет "естественно" ввести ее.
С темной энергией сложнее, ибо неясно что это. Если считать как просто космологической постоянной, то никаких проблем разве что это внешний параметр, которые имеет конкретные значение лишь потому, что так Аллах захотел. Если это действие некоторого поля, кандидатов на которых полно, но они должны по другому проявляться, что сводится к эксперименту для ответа ненужных. Если это вакуумное среднее всего вакуума, то тут куча проблем, ибо не сходится и надо городит кучу лишних сущностей.
Барионное асимметрия частично объясняется в Стандартной модели, которая автоматом включается в теории всего.
Жизнь после смерти Аноним 16/02/23 Чтв 14:47:15 572532 471
Без названия (4[...].jpeg 7Кб, 267x189
267x189
Привет, аноны. У меня будет странный вопрос, но спасибо всем кто на него ответит. Вот среди вас есть химики, физики и представители других веток науки. Мне вот стало интересно.
Существует ли по вашему мнению жизнь после смерти? (В любой форме, я не говорю о христианском представлении).
Какие есть теории относительно существования жизни после смерти, которые опираются на научные данные (знаю только о биоцентризме и о том что ряд физиков объясняют реинкарнацию с точки зрения квантовой физики)?
Ну и вообще было бы интересно узнать ваше мнение.

Спасибо заранее всем за ответы.
Аноним 16/02/23 Чтв 15:10:02 572533 472
>>572532
еще существует сознание с его квалиа
В любом случае, после смерти ты потеряешь память и перестанешь быть собой, так что не парься. Ведь даже если будущее порождение твоего сознания найдет способ восстановить утерянные воспоминания, с его точки зрения, твоя жизнь лишь одна из бесконечности и не имеет ценности.
Аноним 16/02/23 Чтв 15:11:25 572534 473
>>572533
Но вот лично по твоему мнению, жизнь иная возможна после смерти?
Аноним 16/02/23 Чтв 15:54:47 572535 474
>>572534
Жизнь в новом заранее искусственно приготовленном теле - да. Жизнь без тела или в новом природном теле - хрен знает, тут надо сознание изучать.
Аноним 16/02/23 Чтв 17:03:43 572536 475
1.Слышал, для путешествий в прошлое отрицательная энергия типа нужна - а тёмная энергия она же вроде и есть отрицательная. Она сгодится?
2.Хоть ТЭ и расталкивает всё, но ей приписывают положительную массу (причём большей части вселенной), которая по идее всё притягивает. Т.е. сила отталкивания должна и это притяжение нивелировать?
Аноним 16/02/23 Чтв 18:07:58 572537 476
>>572536
Ок, теперь тебе осталось только найти способ равномерно и единовременно накормить этой энергией каждую частицу во вселенной, сущая мелочь.
Аноним 16/02/23 Чтв 18:33:55 572541 477
>>572536
Есть два пути. Первый это превысить скорость света. Второй это белая дыра. Обе истории прохладные.
ТЭ и гравитация противопоставляются друг другу. На начальной стадии гравитация во вселенной доминировала. Сейчас превалирует ТЭ. В какой то момент гравитация может стать доминирующей силой и тогда вселенная схлопнется в сингулярность. Либо тёмная энергия перейдёт какую то величину и вселенная погибнет потому как материя в ней уже существовать не сможет.
Аноним 16/02/23 Чтв 18:46:05 572542 478
>>572541
Тогда в каком смысле у тэ есть масса и положительна ли она иначе нахуй ей приписывают 80% массы вселенной? И если положительна, то она должна притягивать, а уже потом отталкивание тэ перекрывает это притяжение, и мы имеем результирующее отталкивание. Нет?
> Есть два пути. Первый это превысить скорость света. Второй это белая дыра. Обе истории прохладные.
кротовая дыра вроде требует чего-то там отрицательно-мнимого же?
Аноним 16/02/23 Чтв 19:08:10 572543 479
>>572542
Есть наблюдаемый феномен того что вселенная ускоряет свое расширение.Для объяснение это выдвигается теория о существование некой тёмной энергии. Но реальных пруфов существания ТЭ нету. Нету так же понимания что это такое чем это вызванно. Не зная что такое тёмная энергия как ей можно назвать какое-то атрибуты?
Аноним 16/02/23 Чтв 19:13:48 572544 480
>>572536
Плохо ты слышал.
Темная энергия (в самом простом случае как космологическая постоянная, ибо остальные слишком уж маняфантазийные) ни разу не энергия, а из-за принципа общей ковариантности вообще невозможно сопоставить ей какую-либо энергию, импульс и массу. Однако у нее есть вполне конкретный геометрический смысл - она показывается как быстро изменяется масштабный фактор, что отличается от гравитации как кривизны. Грубая аналогия гравитация как искривление показывается как отклоняются мировые линии от прямой, масштабный фактор задает как быстро двигаться по мировой линии.
Те картинки показывают, где показывают процентный состав вселенной, это расчет геометрии Вселенной от реликтового излучения до нас.
Аноним 16/02/23 Чтв 20:23:39 572547 481
2022-08-08 3.28[...].mp4 3052Кб, 640x640, 00:00:24
640x640
>>572512
есть животные которые из отрубленной руки выращивают целый организм
Аноним 17/02/23 Птн 01:52:02 572556 482
>>572544
Можно ли представить себе наше трехмерное пространство как условную поверхности сферы. Сфера раздувается и одновременно на ее поверхности есть бугорки и ямки. Раздувание сферы увеличивает расстояния между всеми точками на поверхности сферы - это типа аналог темной энергии, а бугорки и ямки это типа аналог гравитации?
Аноним 17/02/23 Птн 08:30:05 572561 483
Двач, я тут науки с жизнью перебрал…

Учёные выяснили, что важные для функционированиия клетки организма — такие, как иммунные, например, — для собственного более активного деления заимствуют фрагменты теломер у других клеток. А теломеры, как известно, имеют свойство заканчиваться, и повторно заражённые гриппом лабораторные мышки погибали быстрее, если их не снабжали искусственно выведенными иммунными клетками, снабжёнными запасами этой самой теломеразы.

Теперь поговорим о головастиках. Размножение — крайне необходимая естественному отбору вещь: задействуются на максимум все ресурсы организма — по окончание процесса можно даже в обморок упасть; имеются и зарегистрированные факты летальных исходов от перебора количества "тычек" за раз: от асфиксии, или не выдерживает кардиосистема, сердце (инфаркт), мозг (инсульт) и так далее.

По результатам других исследований, даже в отсутствие внешних стимулов мужские семечки делятся примерно раз в 16 дней. Предположим, что абсолютный максимум, отведённый человеку в природе, равен 60 годам, — т.е., округляя в большую сторону (23 в год), с начала пубертатного периода (когда сперматогенез встаёт на конвейер) у нас есть 1104 "выстрела". Получается, если в процессе сперматогенеза у остальных систем организма аналогично отбираются куски теломер, в течение всего 220 дней регулярного принудительного деления по 5 раз на дню можно либо словить импотенцию, либо за*читься до рака или даже откинуться от старости?!
Аноним 17/02/23 Птн 08:48:26 572562 484
>>572561
>принудительного деления
В реальном мире, мозг просто шлет такие команды нахуи и в каждом следующем "выстреле" просто содержится меньше спермы.
Аноним 17/02/23 Птн 09:37:29 572563 485
>>572561
То есть, NNN это не прикол от джедаев, а всемирная профилактика рака?

Ух6ля, шаблон рвется
Аноним 17/02/23 Птн 09:52:47 572564 486
>>572556
Нет. В первую очередь физическая модель нашей Вселенной четырехмерное пространств-время, вот нельзя просто отбросить четвертое временное измерение. В противном случае у тебя поля разваливаются, да и гравитация не делает, то что должна делать.
Во вторых темная энергия никак не влияет на кривизну, если уж судить по твоей аналогии, то надо брать плоский бугристый лист, а растягивать во все стороны.
Аноним 17/02/23 Птн 12:35:52 572570 487
>>572564
ОК. А если так -- челик ползает по поверхности распухающей сферы покрытой ямками в которые он легко скатывается и с трудом выбирается. Т.е. время тут тоже есть, ведь наблюдатель-челик перемещается в пространстве.
>Во вторых темная энергия никак не влияет на кривизну
Ну так я уточнил, что ямки это не темная энергия, а гравитация, а темная энергия это то что надувает эту сферу и делает расстояния между всеми точками на ней больше.
Аноним 17/02/23 Птн 13:33:57 572571 488
>>572570
В пространстве-времени одни и те же пространственные координаты соответствуют разным местам, аналогия с ямкой не очень рабочая.
А суть ОТОшной гравитации в том, что наблюдателю тяжело/легко откуда-то выбраться, а в том, что он двигаясь по инерции, приходят не туда.
Сфера обладает заметной кривизной, в добавок обладает рядом топологических свойств, которые будут сказываться на локальную геометрию при раздувании сферы.
Еще раз. Темная энергия не влияет на расстояние и углы, она влияет на длину условной единицы в координатной сетки.
Аноним 17/02/23 Птн 13:49:57 572572 489
>>572571
>А суть ОТОшной гравитации в том, что наблюдателю тяжело/легко откуда-то выбраться, а в том, что он двигаясь по инерции, приходят не туда.
Я так понимаю, что суть гравитации это "искривление" путей - искривление геодезических линий (кратчайших расстояний между двумя точками)? И вроде ямки это норм аналогия?
Аноним 17/02/23 Птн 14:01:25 572573 490
ab23171d63e6800[...].jpg 22Кб, 296x300
296x300
>>572571
>темная энергия не влияет на расстояние, она влияет на длину
Аноним 17/02/23 Птн 17:42:21 572579 491
>>572572
>Я так понимаю, что суть гравитации это "искривление" путей - искривление геодезических линий (кратчайших расстояний между двумя точками)?
В целом, да.
>И вроде ямки это норм аналогия?
Аналогия с ямками плохая, потому что поверхность двухмерная, да еще евклидова. Пространство-время четырехмерное и там работают с объектами, которые в малых размерностях вырождены.
>>572573
Есть локальная геометрия, это все про искривление и прочие фокусы, которые задают углы и расстояния.
Есть условно "внешняя" геометрия, это про уже масштабы первого.
Аноним 17/02/23 Птн 20:14:02 572581 492
Аноны, я запутался в физике для яслей, блядь.
Смотрю сериал Пространство и стало интересно посчитать что куда летается при одном ж.
Т.е. есть а - 10м/с2, есть s допустим 30 млрд. метров. (уловные пол пути до Марса).
Я с нуля даю один ж и лечу ровно эту тридцатку.
Какая будет v скорость в конце?
Сколько t времени на это уйдёт?

Школьные формулы требуют либо v либо t, а я откуда знаю, сел да полетел.
Аноним 17/02/23 Птн 20:51:25 572582 493
>>572581
равноускоренное движение
S = vt + at^2 / 2
начальная скорость у тебя 0, S и a известны, t сам считай
Аноним 17/02/23 Птн 20:59:53 572583 494
>>572582
Ну вот я так и понял, начал т рандомно подбирать. Чёт за сутки с небольшим до Марса ебать как быстро.
Хотя постоянное ускорение в 1ж, как я понимаю очень волшебная штука и энергии требует ояебу. Если я нигде не проебался.
Аноним 17/02/23 Птн 21:21:19 572584 495
>>572583
Да проебался, если учитывать разворот и торможение с середины пути. Но всё равно меньше двух суток при хорошем сближении. Прикольно.
Аноним 17/02/23 Птн 22:09:55 572585 496
>>572583
почему сутки - две, вроде бы ж 9 суток.
если быть точным, 774595,66924 секунд.
Аноним 17/02/23 Птн 22:11:38 572586 497
>>572585
я дурной, посчитал для километров
Аноним 17/02/23 Птн 22:42:35 572587 498
>>572586
Значит 2 суток правильно?
Но я всё равно не понимаю почему я должен подгадывать время или финальную скорость вручную. Странные формулы.
Аноним 17/02/23 Птн 23:00:19 572588 499
>>572587
>Но я всё равно не понимаю почему я должен подгадывать время или финальную скорость вручную.
Потому что алгебру не учил?
Аноним 17/02/23 Птн 23:09:41 572589 500
>>572588
Так я не отрицаю, что я долбоёб полный в плане точных наук кого я обманываю, я в принципе полный еблан в любом плане... немного в дизайн могу, на уровне макаки
Просто логика не складывается. Вот типа я на орбите Теры дрейфую, вот волшебный движок с невесомым топливом может жать хоть 15ж. Я говорю компу го на Марс, расстояние известно 60 лямов. Ускорение держи 1ж чтоб я в толчке посидел комфортно, на полпути предупреди перед невесомостью и разворотом, чтоб меня посейдон не обласкал на всю сраку и давай всё то же самое только наоборот, торможение то бишь. 1ж.

А комп говорит нихуя не могу посчитать назовите время полёта или финальную скорость перед экватором. Я ебу блядь? Какой-то странный UX.
Аноним 18/02/23 Суб 03:17:58 572592 501
>>571809
Способность произвести работу.
Аноним 18/02/23 Суб 06:09:39 572594 502
>>571807
Закономерности в движении энергии.
Аноним 18/02/23 Суб 06:39:13 572595 503
изображение.png 39Кб, 600x340
600x340
изображение.png 154Кб, 600x451
600x451
>>571801
В физике материя - это все частицы Стандартной модели и то, что из них состоит.
В более узком смысле, материя это вещество: кварки, лептоны и то, что из них состоит. Различается от переносчиков взаимодействий.

Материя материалистов - это резиновый термин, означающий "все реальное", "все существующее" и т. п. Целиком зависит от смысла, вкладываемого в понятия "реальное", "существующее". Часто оно вообще никак не фиксируется.
Материализмов, таким образом, существует хренова туча.
Аноним 18/02/23 Суб 06:47:44 572596 504
>>571809
В заключение Фейнман дополняет свои рассуждения следующими словами: «Очень важно понять, что мы в современной физике не знаем, что такое энергия. <...> Есть формулы для расчета числовых значений, и, если их сложить все вместе, получается всегда одинаковое число».

https://elementy.ru/bookclub/chapters/433640/Fizika_rasskazannaya_na_noch_Glava_iz_knigi?ysclid=le9en3nusd393873986

https://www.youtube.com/watch?v=lTZsFfs7p9k&list=PL8YZyma552VcqcCVofipK0VUwWqHqfiFF&index=3
Аноним 18/02/23 Суб 08:39:59 572597 505
Кто такая Франсвильская биота?
Аноним 18/02/23 Суб 10:59:01 572601 506
>>572597
мразь которая спамит в доистории
Аноним 18/02/23 Суб 15:16:56 572604 507
Настройки X
Ответить в тред X
15000
Добавить файл/ctrl-v
Стикеры X
Избранное / Топ тредов